WALL-E/Headscratchers

Everything About Fiction You Never Wanted to Know.


WALL-E

  • The need for a Holo-Detector just really bugs me. First off, its this gigantically lavish machine that's ultimately useless. I mean think about it. You put a plant in there, it guides the ship right? But couldn't they just use the coordinates that were programmed into EVE or her spaceship originally? Also the whole "guided by plant" thing is just hilarious to me- what if someone thought it was a trashcan and tossed in some leftover salad?
    • There are no salads in the future. All the food is recycled organic matter in a cup. The Holo-Detector is a relic of the past, intended to be a fanfare-like machine where the passengers would congregate to celebrate their return home, most likely featuring the CEO of BnL triumphantly putting a living thing people have only seen in the form of holograms and imagery into the machine. Obviously that part of the manual wasn't elaborated on.
    • Presumably, the Holo-Detector would know the difference between a "living" plant, and one that has already been sliced/ripped from the ground. The need for a plant is just a confirmation that life is sustainable on earth again, so as to avoid false positives
    • The Holo-Detector wasn't guiding the ship there. Obviously the ship knows where Earth is. It's not a question of location, it's a question of determining whether life is sustainable there. The Holo-Detector can probably tell the difference between a plant grown in the (very likely) extremely controlled environment of the Axiom and one that was grown on Earth. Don't think of the Holo-Detector as a guidance system; think of it as a lock, with a plant from Earth needed to open it.
    • But that seems needlessly complicated then. Why would they need to actually have to transport a real plant from Earth, across space, to the spaceship and hope that it doesn't get destroyed along the way when they could have arguably just direct the ship whenever they needed (well before Auto took control)?
      • Because the whole purpose of that thing was to corroborate that life was sustainable on Earth again.
      • Did you not watch this movie? EVERYTHING is needlessly complicated. Why have a robot that types thing via hunt and peck? Why is the Autopilot separate from the onboard computer? On and on and on...
        • "needlessly complex" can also translate to "modular" and/or "decentralized." Having the autopilot separate from the ship's systems made it possible to have the autopilot disconnected in the event it went bonkers without half-gutting the ship. Extrapolate this to all of the ship's systems.
    • Also the Holo-Detector was retracted into the floor and the captain needed to activate it from the bridge so that somone would be able to put a plant inside of it.
    • The Holo-Detector was also in a centralized section of the ship, and emerges with great pomp and circumstance. It's there for ceremony as much as anything.
  • How does the plant that Wall-E find survive for so long? It is manhandled, taken into space for weeks/months on the trip out to the Axiom, exposed to the vacuum of space for at least five minutes, wrestled over, and the worst that ever happens is a leave falls off?
    • I'd imagine that EVE was specifically built to store a plant, so I have no problem with that part. Her compartment has lighting, water, air circulation, etc. The hard vacuum exposure is a little harder to reconcile, however.
      • Scientists in real life are growing plants in a vacuum and in lunar style conditions, if that helps any.
      • It's only actually in vacuum for a few seconds, and it's obviously a really tough plant from the fact that it grew in the first place.
  • In the ad for Axiom, several ships were shown blasting off, and a "fleet" was referred to. Yet we only ever see one ship. What happened to all the others?
    • They're still out there in space somewhere, if the rest of the autopilots are anything like the Axiom's AUTO.
    • Unless the "pods" seen moving about in the end credits are presumed to have come from other ships. There appeared to be only the one dock, so the others might have needed to actually use their landing pod/shuttle things.
    • We never know what city we see for most of the movie - there's likely ships and docks all over the planet. Furthermore, it's likely that all of the other ships would be programmed to return home as soon as one ship activated the conditions necessary for it. We just don't see them.
    • Axiom is the flagship of the fleet. Presumably if it returns it can call the others back. Remember, A113 was directed specifically at AUTO.
    • Perhaps it needs to find Earth again because everything in space is constantly moving very, very fast. The Sun goes around the galaxy at over 918,000 kilometers an hour, dragging the planets along with it. They've been floating out in the middle of nowhere for 700 years, so between the time they left and the time they return, the sun would have moved about 563,291,128,000 kilometers (that's five hundred sixty-three billion, two hundred ninety-one million, one hundred twenty eight thousand kilometers), Give or take a few, of course.
      • In the Castilian Spanish dub, the CEO refers to the automatic pilots-any word on other languages?
        • I believe it's the same in English -- AUTO was just one of an entire class of auto-pilots, and they were all given directive A113. I personally prefer to imagine that all the other evacuation ships are still floating out in space, ripe for a sequel, or developing into a bunch of Planet of Hats-like isolated communities, or waiting for one of them to make it back and result in a Culture Clash with the descendants of the Axiom.
    • They were scattered all over the universe. A bunch of them landed on Planet Gunsmoke and began Trigun. Also a particularly big one was the setting for Last Exile. A few more fell into a wormhole and fell into a long, long time ago in a galaxy far, far away and so explains why humans are in Star Wars.

Of course, my mistake.

  • Where did they get the seeds from the end credits? It looked like plants were much rarer than what they seemed to have rather quickly.
    • At the end of the film the camera pans out from the city for the first time to show that outside the city a bunch of plants are growing.
      • I got the impression that that was a farm, set up by the recolonizing humans. Plus, even if it was already there, it doesn't explain how they got enough to grow food crops like corn (which we saw), and not just weeds. Corn, as I recall, is incapable of reproducing without human aid, and as such using seeds from god-knows-where is the only possibility.
        • Given that the stated purpose of the starship fleet was to recolonize Earth eventually, they probably had stored supplies for that purpose. With very good preservation techniques, yeah.
        • Actually, I got the impression that all those plants had simply grown in the time Wall-E was away from earth. The Axiom returns to earth at lightspeed, yes, but he probably spent weeks, if not months hanging onto that Axiom-bound rocket, especially considering it passed the sun..
          • ^^^^(reply to above) I doubt this aspect of time dilation would have been considered as the movie's take on space travel is clearly designed for expedience and beauty. I got this impression from when the probe ship Wall-E hitched a ride on passed all the beautiful sights of the solar system in what we are lead to believe was a very short, scenic period of time. This itself is probably related to the joke that sci-fi authors have no spatial sense of actual space (for example it would be near impossible to hit an asteroid in the asteroid belt, but its commonly depicted as a death trap in fiction), so this leads me to believe they would ignore any realistic temporal-spatial problems like that.
        • Given that that was not a warp drive-driven rocket, it may have taken years if we assume that the Axiom is a good distance outside of the solar system.
    • From the Doomsday Vault.
      • It's entirely feasible that the Space Cruisers were equipped to get Earth back to first base in terms of a stable ecosphere. So, you're probably looking at DNA logs and plant producers, cloning machines, and what-have-you. AKA the Doomsday Vault.
      • They used a Garden of Eden Creation Kit.
    • The problem with earth was toxicity making plants unable to grow. It still sounds far fetched, but it that was the problem, then I guess the ships were initially equipped with seeds in a doomsdays-vault-esque sort of way until earth is restored.
  • How did they move so well? With no real muscle mass to speak of and bones horribly shrunk, and generations in microgravity, why weren't they utterly paralyzed by full Earth gravity?
    • That looked like full Earth gravity on the ship to me. Those chairs they were in hovered, as did some of the bots, but other than that, standard gravity seemed to be in effect.
    • I think it's implied that "the effects of microgravity" was just a convenient excuse they hid behind to avoid having to own up to the fact that they'd never done anything. Remember, it's only Shelby Forthright who mentions microgravity, and he's not the most reliable source.
  • Where are the old people? We see adults on the Axiom. We see babies. I could even be persuaded to accept that some of those blobs were teenagers. But the elderly? Where are they? Is cosmetic technology so good in the future that everyone stays looking like a young adult their entire lives? Or are these people so fat that they are dead by middle age from heart attacks?
    • There are old people on the Axiom. Look for the ones with gray hair and perhaps bald spots. They're shown in a few shots, and I'm guessing you'll find some blurred in the background. I'll also presume that you were being snarky and cynical with the 'heart attack' comment, since the plaques of the previous captains of the Axiom show that humankind managed to extend their lifespan to at least 120 years.
    • Ever see a wrinkled pig? No sir!
    • Look just behind the captain when humanity is exiting the Axiom for the first time. There is a little old lady with grey hair and a walker.
      • But wouldn't a walker imply that she could actually ... walk?
        • I think it was just another fatty using a robot as help to walk, which ties in with the overall ending theme of humans and robots working together to recreate the biosphere.
  • Shouldn't most of the soil still be too toxic to grow plants in? Wall-E's plant only grew because it was in a sealed environment.
    • For that matter, how DID Wall-E's plant grow? It couldn't possibly photosynthesize in a closed, lightless refrigerator.
    • This may be the most literal application of Fridge Logic ever.
      • More then likely the plant was a plant that require fresh soil and not much sun light, that and the fact it was only a small plant it must not have had been there for too long. That and to answer the original question, if the plant grows then the soil isn't toxic anymore, the fridge's bottom was probably rusted out.
    • The refrigerator door may have initially been open, allowing enough sunlight in to let the plant grow. It's not beyond the realm of possiblity that one of the massive sandstorms could open or close the refigerator door.
    • The door wasn't actually on a fridge when Wall-E found the plant- the fridge is resting to the right of the door, and the plant is behind the door. So, apparently the fridge door came off much earlier, and the plant grew behind the door.
      • That, and some plants are incredibly hardy little buggers. Not necessarily, however, the same ones that are edible to humans (and even the ones that are will become a nice collection of toxins that are harmless to plant-based life, not so once unleashed in an animal's metabolism). One random type of plant springing up wild does not a garden make...
      • When I was in fourth grade, our teacher planted a seed in a pot in a closet as part of our science lesson. It actually grew a lot bigger than the one in the movie before it finally gave up the ghost.[1]
  • How did they reproduce when they lived on hoverchairs too small for two, and they weigh too much to be on top of one another?
    • The technology to do this appeared in Brave New World. You harvest the eggs and sperm from donors, fertilize them in a test tube, and grow the embryo in a "bottle" that mimics a human womb. With the kind of technology the ship had, this isn't too far off.
    • That's the most likely considering none of those people ever touch another human being.
    • It also explains why we don't hear any women shrieking the Stock Phrase, "Oh no, my baby(ies)!" when the babies are in danger and John and Mary have to rescue them.
      • Although, what Mary DID shriek when she prepared to use John as a babycatching whip was priceless. (although, of course, it also ignores the fact that she'd have no strength in her shoulders for such a move)
    • Although that raises the question of how they reproduced after leaving the Axiom, with no one to teach them the facts of life.
    • The Captain basically had the internet on the ship's computer with a voice interface. If the survival of the species depends on the smokey voice of Sigourney Weaver giving a detailed explanation of the human reproductive process... well, I wouldn't complain.
      • People have been having sex for far longer than there's been Sex Ed. It's just something we do.
      • Yes. Let's assume that the humans on the ship know how to masturbate (especially in an environment where the only thing to do is hang out in a chair, and where no one is really paying attention to each others' forms of amusement, etc. Also, is cyber-sex out of the question here? Everyone is communicating over instant messenger, aren't they? And lastly, is there still pr0n on whatever internet they're using?). Once the humans returned to earth and started spending a lot of time together, it probably wouldn't take long to figure out how to get it on, even if they DID have no previous knowledge. All a man and woman (or, er, any two people, genders aside) would have to do is compare masturbation techniques, and think, "Hmmm ... well, MAYBE we could do this at the same TIME in some sort of interlocking motion ..."
        • They've got video conferencing, in real time... far superior to what we have today. Unless you've got a thing for overweight people, are you really going to get yourself off in a cyber?
          • Remember that our "overweight" is their "average." There's no reason why two given members of this society should find each other unattractive. Even if they have historical records to compare themselves to, their definition of beauty will be more strongly influenced by what's normal to them. This is true in our society as well - we know that heavier women were once the standard of sexy (Tevye wants his wife to have "a proper double chin"), and that a lot more attention used to be given to a woman's legs.
            • Never mind whether they find each other attractive or not -- there are physiological barriers to a very-obese man and a very-obese woman engaging in intercourse, even without the issue of bone loss. Roseanne Barr once complained that she and her husband had gotten so fat that they could no longer have sex. In WALL-E, this now applies to the entire human race.
      • Thinks about writing something really inappropriate. Doesn't. I'm a terrible person.
  • And speaking of technology, we know for sure that the Axiom has a hyperdrive. They also apparently have some sort of nebular-hydrogen-fed atomic assembler to still be able to toss large amounts of trash out into space even after 700 years. Wouldn't this sort of tech make garbage disposal and re-terraforming Earth a snap? I suppose it serves to underline that the problem facing the passengers was one of philosophy, not engineering.
    • One could also fanwank that fabricating matter out of high-energy hydrogen is a lot easier than recycling low-energy middle-of-the-periodic-table elements. And of course, fixing the ecosystem is a whole different nutshell from just the raw-matter, too-much-waste perspective.
      • Too much effort. Simpler answer: once Directive A113 (abandon Earth, forget it) was finalized, Buy and Large explored new sources of raw materials. Plenty of those around if you have interstellar travel tech and robotic labor. Asteroids for minerals and metals. Robotic farm ships for food. They've just strip mined the asteroid belt same way they originally strip mined Earth.
    • And speaking of hyperdrive, how is it that the probe ship can travel at a leisurely speed from Earth to the Axiom in what seems like a matter of hours, yet the Axiom has to use FTL travel to get back to Earth? How long did the probe ship take to get back? And how many centuries did the cockroach wait for WALL•E to return?
  • Why weren't any WALL-As used in Earth's cleanup? Their bigger size and greater capacity for garbage probably could have helped speed up the job of cleaning Earth.
    • Presumably, the WALL-As weren't invented until after the Axiom had long left Earth.
      • And because of the fact that the Autopilot of the Axiom was ordered to never go back to Earth, the Wall-As wouldn't be able to clean Earth anyhow.
        • Also, the name WALL-A (A standing for Axiom class) suggests that they were specially created for the conditions of the Axiom, and Wall-Es for the special conditions of Earth. The Wall-A's are good for large areas with large amounts of garbage and nothing that can be damaged. On Earth, the planet that people originally planned on going back to, they'd want robots that wouldn't run over the last couple of things with worth.
          • There were bigger WALL-E units. WALL-E lived in one. They just weren't working for some reason.
            • No, WALL-E lived in a storage unit for him-sized WALL-Es.
    • Either that, or it's because WALL-E's smaller size makes it easier for them to move around and get into small spaces to clean stuff up.
    • Also, the exact reason for the failure of the WALL-E units is never explicitly stated. For all we know, the WALL-As could have failed for the exact same reason.
    • Bigger isn't necessarily better - Off the top of my head, moving something the size of a Wall-A is a tricky maneuver at best(they'd likely sink in the literal sea of trash covering cities), they don't seem equipped to stack trash cubes more than two or three high(they couldn't build towers as tall as the Wall-E's could), and trying to run them on solar power would have been difficult(they would need proportionately much larger solar panels). And the list of potential behind-the-scenes issues is a lot longer.
    • Watch the movie carefully. The WALL-As do not have tank treads like WALL-Es. They actually move on tracks built into the floor of the Axiom like trains. They would simply not be able to move on Earth until tracks had been built for them, which eventually happens by the credits.
      • It is also implied that the Wall-Es simply were not good enough for the job, the main one only survives because he is able to scavenge from others which perhaps also implies that there were simply no replacement parts for others so as they started breaking down the only ones capable of surviving were the one's who would scavenge from others.
  • The Axiom's stated purpose was to be a "shelter" for humans for a few years while the WALL-E units cleaned Earth up. So where are they getting enough food/water/fuel to last for seven hundred years without arousing suspicion?
    • They're in space. They can move faster than light. Space has stuff in it. Do the math.
    • Soylent Green?
    • Wondering how they eat and breathe and other science facts huh? I think you need to hear the MST3K Mantra
    • They probably have some sort recycling system onboard that turns waste into edible mush... in a cup!
      • WITH A BOX OF SCRAPS?
    • One of the systems in the captain's "briefing" just after he tells AUTO that he still has to follow protocol is a regenerative buffet system--so, yeah. What the guy above said is canon.
      • I am going to try SO hard not to think about that.
        • Oh come on, just think about it: that's what all food was at some point.
    • Besides, if the population is kept constant, the total amount of biomass on the ship will be nearly constant too, which means that with a sufficiently efficient recycling system, you could have enough food for a very long time. Water is the same. Fuel could be a problem, but as we can see from EVE, lots of energy can be packed quite compactly in the future, and solar energy can be used. But what does bug me is that they're seen jettisoning huge amounts of garbage into space, where recycling it would make so much more sense on a ship that has already lasted for 700 years and is planning to do so for a good while longer.
      • Considering waste and a lack of recycling is what made them abandon the Earth to begin with, I thought the fact that they moved their garbage-spewing to the universe as a whole was a nice touch.
    • Look at what they have. Is it so unlikely that they have energy-to-matter converters?
      • Yes. Besides the Sci-Fi Writers Have No Sense of Scale aspect (It takes nearly 3 Gigawatt-hours of energy to produce a single gram of matter if you could convert 100% of energy into matter), there are conservation laws that forbid you from doing that. Conservation of baryon and lepton numbers require that if you convert energy into matter, you must also produce an equal amount of antimatter.
    • What's to prevent some of the other scout ships from raiding nearby planets? This would not violate the directive to stay in space as only the robots doing the harvesting would leave and return. Any plant life brought back to the Axiom would be classed as necessary to feed Axiom's people, so AUTO would be allow it on board to be processed; he doesn't have any reason to tell the captain what he's up to, and no one else notices. Of course AUTO would consider this part of his duties to provide for the humans. AUTO's main plan is to keep the humans away from Earth, so he is specifically watching to eliminate any Earth plants the EV Es find, any others are just food.
      • Also explain's M-O's obsession with "foreign contaminations"; one of his duties is to keep any planet dirt confined to the ship's processing area.
  • Seriously, what is with the Axiom's economy? Buy 'n' Large is obviously charging for food, clothes, etc; and, thus, making money...but they control everything on the ship, so there's nothing to spend the money on. And how are the passengers making money if nobody does anything, ever?
    • Again, I suggest the MST3K Mantra. In fact, considering that the Axiom is a cruise liner in space and the Satellite of Love is more or less a ship in itself I suggest all questions about the Axiom and daily life be relegated to said mantra, as we should really just relax considering how they've been living up there for 700 years, they just have.
      • If you want actual reasoning, no matter how thin, then the voyage was probably all-expenses paid to begin with. After all, survival of the human race versus a five-year voyage on a space-faring shopping mall? Buy N Large probably wasn't that terminally retarded. And if that doesn't work, when Directive A113 was activated, it's not a stretch to think that everything then became free. Terminal space voyage on a space-faring shopping mall with no jobs'll do that.
        • Wasn't that technically Jossed when the Captain mentioned the "free septucentennial cupcake-in-a-cup"? If everything had been free for years, why bother specifying that that in particular was free?
          • Double extra technically, doesn't there have to be a downtime of some sort in order for there to be a Jossed situation?
        • It was most likely a scripted speech, and since the Axiom is sponsored by BnL, it was probably just a buzzword that caught on and they never bothered getting rid of.
    • My personal vote is that even before they left Earth, they were in a post-scarcity society, and money no longer had any role besides being a cultural hang-over/way of keeping score. The people on the Axiom either get an allowance, or 'earn' money through playing games or whatever.
      • Or they're paid for watching ads. Which would be financed by BnL. Mystery solved?
        • Elegant, I like it. Considering that you can actually make money doing that right now (filling out surveys, testing new products, pence-per-click, etc etc) then that seems a perfectly logical solution.
        • There's no sense in paying them for watching ads, or use advertisement anyway. BnL is a monopoly with total public awareness. They are being taught about in schools, for cripe's sake. I think the billboards and jingles are there because humanity has gotten used to them, and maybe advertising BnL is the only form of art left (think propaganda posters... or Demolition Man).
        • Dammit! I was going to say that!
    • We also don't know that the residents don't do anything. I know I would be bored out of my mind in a place like that without anything to do. There probably are jobs to be done on the ship that we don't see, and just because robots can take care of everything doesn't mean that they do.
      • But... the whole point was that they were all insanely bored and useless precisely because they never did do anything, and because robots did do everything for them! If there were jobs doing after all, and if robots didn't do everything, the passengers wouldn't be as helpless and sedentary as the movie went out of its way to portray them as being. Also, they are passengers on a cruise ship. They are supposed to be just relaxing for their five-turned-seven-hundred year cruise, while the robots cater to their every whim... That was, again, the whole point! I'm going to go with "post-scarcity economy" here, because it was so heavily implied by the movie: The robots do absolutely everything, and the humans just relax and get bored all day long; either they get "allowances," or everything is free (with specific uses of the word being just tradition, or an obsolete buzz-word).
    • In the background of one Axiom shot, a huge billboard flashes "BUY NOW - PAY LATER." It could be that people were meant to pay (at interest) after they went home, but since they weren't going home, the actual payment never came up.
    • How about. BnL owns the ship and sells them stuff. But the passengers all own shares in BnL and the income BnL raises from sales on the ships are paid back to the passengers as dividends.
  • Why would WALL-E need a tape recorder built into him? He's a trash compactor; why would he need to record sound?
    • Well, the little guy is obviously built to have some sort of speech and sound recognition wired into his AI, probably to give him a friendly personality to any human he should come across. Maybe the sound recorder was put on so that he could transmit messages. Or maybe he wired it into himself (he's shown himself to be quite competent at fixing himself, as well as very aware of his inner workings) so he could have a little bit of Michael Crawford everywhere he goes.
    • For the same reason you need a fridge with internet access.
  • Regarding the above: I have no real problem with WALL-E having some sort of audio playback/recording device, but why a tape recorder? Remember, the WALL-E droids were apparently rather state-of-the-art when manufactured. Tapes are considered obsolete in the present, and WALL-E was supposedly manufactured centuries one century from now.
    • Simple, he probably added it on himself after a couple hundred years, so he could listen to some tunes on the job- we don't get a close enough look at the other WALL-Es to see if they also have buttons like that. If he added it himself, he'd be restricted to what he had on hand at the time, and he probably had just found a mostly intact tape recorder at juuust the right moment.
    • Actually, I've seen the film twice now, and it's never really revealed to be a tape recorder specifically, just a generic audio recorder/playback system. All we ever see of it are the control buttons, and--let's face it--the "pause" "stop" and "play" buttons have had the same universal symbols for decades now.
    • In fact, it's probably a hardware digital recorder. Reviewing the facts: 1) he can set it to record at any time, at any point, just by pushing record. Tape players require you to push record and play. 2) When he gets almost terminally crushed, there's no magnetic tape anywhere in his body. 3) After he's repaired and a number of components are replaced, Eve pushes the play button...and there's only static. If there was a tape, we would have seen it get pulled out.
    • The "tapes" obviously aren't normal magnetic tapes. EVE pulled out the track at one point, and she's electromagnetic enough to light bulbs by touching them, and it worked just fine. It's probably just retro for the sake of it.
      • Also, he never has to rewind the thing.
      • She's only electromagnetic by choice. The lightbulb only lights when she thinks about it a moment. EVE's telekinetic...?
      • The video cassette of Hello, Dolly! was very definitely supposed to be a (magnetic) VHS cassette. That also didn't get demagnetized when EVE touched it, implying that her light-bulb-lighting field isn't always on.
  • Why does AUTO manually press buttons on the bridge rather than controlling the ship's internal circuitry itself? In fact, why do all the other robots have to push buttons?
    • It makes the humans feel better. I'll bet he's just pushing the buttons for show, and actually controls things through the circuitry anyway.
    • Unless it's a safety feature. He obviously has some accommodation for if he were to go rogue (the manual/automatic switch, notably), so maybe this is a safety feature? As the eventual ending shows, it's much easier to fight a robotic AI if it has the same problem controlling the ship (i.e. needs to actually hit input buttons) that you do.
      • However, he wasn't touching anything when he made the ship tilt at an angle. So it's probably a case of 'making the humans feel better' as mentioned above.
        • He's a steering wheel, meaning all he has to do is twist himself. Notice that it's the first thing the captain does once the ship's on manual
      • Wasnt the ship-tilting a result of AUTO spinning to throw the captain off of him? That was my understanding.
    • This was probably just done so it was obvious to the viewer when he was doing something. Just holding still and making things happen doesn't have the same effect, you know?
    • Button-push interfaces were designed so that a robot operator could be easily replaced by a human one if need be.
  • Why are the Axiom's escape pods armed with a self-destruct mechanism?
    • Who said that AUTO or the security bot couldn't have rigged something up, and, due to some sort of user-friendly subroutine, labeled it "SELF DESTRUCT"? Or maybe the ship designers were just sci-fi geeks and included it for fun, like in their favorite classic movies.
    • So, after some thinking, this troper thinks he has a reasonable explanation. He suspects that the self destruct mechanism was armed by GO-4 (he is assuming here that most other people initially assumed that WALL-E accidentally armed it, based on the admittedly flimsy logic that he did) in order to carry out directive A113... by destroying the plant. Going based off of this, it's not unreasonable to assume that the self destruct mechanism was locked behind some kind of security system that only a security bot like GO-4 could access. The function is mostly likely intended to destroy materials of a highly... sensitive nature. Perhaps Shelby Forthright had some 'personal demons' he wanted to 'exorcise,' as it were, and included it as a standard feature so as not to raise suspicion (this, of course, assumes that somewhere there's a person who would/could piece something like that together... or, at least, there was when Earth was abandonded). This is 90% speculation, of course.
    • A theory of mine states that it was installed for possible weapon use. It is a space ship after all, and you would think there would be some way to defend itself. However, in the film we don't see any weaponry of any kind on the ship (the fact that it is a passanger ship certainly attributes to that). That being said, rather than add on cannon systems, they just installed self-destruct mechanisms. So, if you shoot one of the pods out, and it lands inside a nearby ship. And self-destructs, well...
    • The escape pods were capable of space travel. That makes them space ships. And as everyone knows, all space ships need a Self-Destruct Mechanism!
    • Building on the above troper, it would only be natural for a space escape pod to have a self destruct. Imagine it's damaged and rockets off course after the Axiom is destroyed. Without any way to return to a habitable world, the inhabitants would probably enjoy the option of blowing themselves up rather than popping the airlock or waiting to die of thirst.
  • When it's revealed that AUTO is secretly trying to keep the humans from returning to Earth, he uses a little electric-shock device to shock WALL-E in an attempt to get him out of the way. He later attempts to use the electroshock device against the CAPTAIN! Secret agenda or not, who would weaponize a steering wheel?
    • Probably so that it prevents others from controlling it, sort of like self-defense.
    • Alternatively, AUTO could have modded itself. It did have total control over the robots, which presumably includes maintenance bots, who could do something like that.
      • Partially incorrect. He had total control over the security robots. EVE and pretty much every other robot that was native to the ship were just as unhappy with AUTO's decision as the humans, if only because they were programmed with that kind of ideals. So that means that any robots not involved with security did not have to answer directly to AUTO.
      • Also, it prevents rogue robots from blowing it up. Remember the massage bot? That was probably not an isolated incident.
      • Actually, if you look at his profile on the official website, it's apparently a design feature.
  • How did the tape of Hello Dolly last that long? Magnetic tape degrades over time- it should have been too damaged to use by the time the ships went out, much less 700 years later.
    • Maybe Wall-E figured out how to make new tape and copy it.
    • Or is has degraded and that's why it only plays the two songs(Put on Your Sunday Clothes and It Only Takes a Moment) skipping a huge portion of the original film. And yes that really bother this troper.
    • Or it wasn't a traditional tape - it was a much sturdier one made as part of a retro fad.
    • It was a YouTube clip!
      • Tape was actually made of some form of phlebotinum but aesthetically designed to look like a retro VCR cartridge?
        • Probably. Given that the media on the tape was compatible with an iPod, I'm guessing that it was probably some kind of modern incarnation of a tape that was designed to be more durable.
  • Here's a good one: AUTO's stated purpose after Directive A113 is to prevent humans from ever returning to Earth. Why, then, would the EVE probes ever be sent out in the first place? It's fairly obvious that no one on the Axiom had any idea what the deal with them was, and it's even more obvious that Eve was supposed to not find anything. So, why send the probes out at all? And if you do, why send them to Earth? Why not send them to Mars or something?
    • If you were paying attention, AUTO said something about annual reconnaissance, so sending the EVEs out was a yearly thing. At first, he probably kept sending them to make sure the humans didn't suspect anything was wrong. As time went on, the people eventually forgot what the EVEs were really for.
    • I figure there are two options. a) AUTO doesn't actually have full control over everything on the ship, and was just unable to stop the probes from being sent out. b) Whoever coded the response to A113 did a crappy job, and it didn't override the earlier directive to keep sending probes. Because of this, AUTO kept doing both until they came into direct conflict.
      • It's already been firmly established that whoever programmed AUTO didn't do the best job (hence why the captain was able to access the "classified information" just by giving a direct order, but not to override the A113 directive), so inconsistencies in its actions aren't implausible.
      • A113 was initiated on the assumption that Earth was too far gone ecologically speaking. There's no reason not to keep sending the probes even in that situation though, even from AUTO's perspective. If AUTO is operating on a correct assumption (that the earth would never recover), it has nothing to fear from the probes. They're already paid for, why go to the trouble of turning them off if they aren't expected to have any consequence, AUTO isn't evil. The glitch one would assume comes in where AUTO is given conflicting directives that it's not equipped to deal with. From AUTO's perspective, the safer option to protect the Axiom's cargo is to continue with A113. New data that contradicts A113 is therefore rejected and destroyed. Sure, more prescient programmers would have accounted for this, but by the same token a more prescient society wouldn't have made the planet unlivable.
  • The status quo at the end of the movie is as follows: two living animal species, one plant species, and however many bacteria. And robots. Where did they get all the species from? Suddenly, in the ending credits, the ultimate genetic bottleneck has transformed into massive farms and gardens.
    • Doomsday vault. See above.
  • Three words: "regenerative food buffet" as said by the Captain during his morning coffee. How does THAT work? I can accept a Doomsday Vault and test-tube babies, but how far does the Mystery Science Theater 3000 mantra go?
    • Star Trek-styled replicators, that's the only logical explanation. That may have been the start of their problems back on Earth too, if they made stuff in excess . . . given BnL, that probably happened.
      • That would explain why their problem was tons of excess trash and not, you know, completely depleting Earth's resources.
    • "Eat recycled food. It's good for the environment, and ok for you."
  • Why, 700 years in the future, are there fire extinguishers that look exactly like the ones we have today? You'd think they'd have improved the design or simply made them automatic, especially on a ship as impressive as the Axiom. Given, it was an escape pod, but still...
    • Well, the ship was designed in late 21st/early 22nd century, so the standards could be the same as the present; or they just like retro.
    • If it ain't broke, don't fix it. The basic fire-extinguisher design has been in use for centuries for a reason.
      • ...and that reason assumes the presence of human operators that are capable of using it-which is to say, capable of lifting it from its storage hook, approaching the fire, and holding on while the nozzle deploys a high pressure stream. The Axiom's inhabitants couldn't even do one situp to reach a beverage cup...
        • Well yeah, they can't now, after generations of microgravity and hoverchairs. But the escape pods, like the rest of the ship, were designed for use by the generation that left- especially the escape pods, given that unlike the in-ship equipment, it would have seen no use in the past 700 years. They simply never bothered to replace it when they got fat.
        • Remember the manual door release? That was even harder to operate than the fire extinguisher. It's the same principle as above - probably designed to be used by general-purpose assistant robots anyway.
      • Also: Interesting how the retro fire extinguisher manages to hold enough propellant for dozens of seconds of thrust... despite being included in an escape pod. Just how big of a fire are you going to put out in there? Also! The exhaust plume behaves in a wildly inaccurate manner. In a vacuum, the plume would spread out to be much wider, and would continue moving after leaving the nozzle, with no air friction to slow it down. In the movie it behaves like the plume left behind from an atmospheric rocket launch, which is pretty, but not especially realistic. In this romantic children's movie about talking robots.
  • So at the end of the movie, EVE and WALL-E are living happy, the humans are back on Earth, and the Captain is enthusiastically leading everyone to a bright future in which there's enough "vegetable plants! And pizza plants!" to go around. Here's a silly question: what about the nasty sandstorms? Could someone please just reassure me that they manage to find shelter in time instead of being totally wiped out after the credits, so I can sleep at night?
    • They survived the sandstorms until enough vegetation existed to hold the soil together the same way they had enough food to survive until they had enough plants to feed them all- they probably used the ship as a shelter until they had cultivated enough for the planet to be livable.
    • I thought the "sandstorms" were actually just the result of EVE pods landing on Earth. Once the humans came back to Earth, the sandstorms would have stopped because there wouldn't be anymore pods landing there.
      • Aren't the sandstorms caused by WALL-E's massive garbage towers falling over?
      • What are you basing that off've? They don't seem to be at all associated with the pods, happening well before and well after they land.
        • The ship landing at the end of the film kicks up the same kind of sandstorm as seen earlier in the movie. This troper thought that was a hint that the storms were not natural. Since multiple ships would have sent down EVE pods over time, that would explain the many different sandstorms at different times. This troper is pretty sure that the storms would have stopped because eventually the other ships would be recalled - Operation Recolonization would be rather useless if there was no way of alerting the other ships that the planet was safe again. So...sleep well, okay? Everyone's fine.
          • Er, yeah. You know, a mere few decades ago, a massive agricultural screwup resulting in the loss of most-but hardly all--key plant life occurred on the American prairie. The result was massive erosion and dust storms, with dust kicked up by any wind without the benefit of trees to disrupt the windflow or grass to hold the dust down. Today, we call it the Dust Bowl.
      • On a similar note, I thought back to the Axiom's landing sequence -- the thing casts a shadow over the ENTIRE city as it lands, and still towers over every skyscraper. Not to mention it parts the sky and blows away a whole lot of dust. Granted, it never explicitly states that it ends the sandstorms, but that was my best interpretation.
    • They still have the Axiom, don't they? They can use that for shelter until they finish constructing some independent buildings.
      • A brief shot in the credits shows a group of people using the escape pods as shelter while they cook around an open fire. If they can survive re-entry (as EVE didn't seem to worry about Wall-E's safety when she shoved him in one to send him back to Earth) they can probably survive a dust storm or two.
  • Why was EVE trying to kill anything that moved in the beginning?
    • Self preservation. Also, she was a crazy bitch.
    • She was scanning for plant life. If it moved enough, it probably wasn't a plant, and therefore, killable.
    • Also, remember that not everyone got off the planet. there were probably few years where humanity on earth survived Mad Max style before they were all finally wiped out. In those years, they might have lost a few EVEs to vengeance-seeking mutant mole people, and they needed big honking lasers to defend themselves.
      • If vengeance seeking mole people can survive, what is the point of the EVE probes in the first place? Earth would then obviously be surviveable and not toxic.
        • The ark was a better option than letting 99.9% of humanity die and the few survivors turn into mutants who would eventually die anyway. The mole people phase was just the poison driving people insane before it killed them...
        • Also, one should note that there was a cockroach around. Animals and other moving things could probably survive for centuries by eating the garbage, but this is not a sustainable lifestyle for humans, I guess. Plants indicate the ability to start a real ecosystem again.
    • Rule of Funny, guys.
    • Because AUTO had reprogrammed her to be trigger-happy, in the hopes that she would "accidentally" destroy any plants she found before she had a chance to collect them.
  • Remember when they were outside the ship and WALL-E handed the plant to EVE? Sure, it may have been for a few seconds, but even still, how did it survive exposure to the vacuum of space?
    • Temporary exposure to vacuum isn't actually instant death; there's no reason a plant wouldn't survive with nothing worse than a bit of dehydration.
    • Plant life would be able to survive the vacuum of space a lot better than animal life could.
      • Err, isn't this the same as the first IJBM on the page? Check that out for more info.
  • Where did all the water go? Where did it come from when it came back?
    • Climate change?
      • Theres still that pool on the ship
        • The water didn't "go" anywhere. If the infrastructure would collapse now, many cities and many crop fields would became desert, since the natural growth of plants was disrupted and having mountains of trash wouldn't help, either.

The water might have gone to the natural places where it should go: oceans, rivers... The harbor shown in the movie might have been an artificial one, higher than the planetary zero level of water and kept wet only via dams and water gates. No humans to operate them meant the water slowly went away and the ships slowly sat in the sand. Actually, there is a scene where some oil ships fell down like dominoes after one of them is hit by Eve. That might suggest that the ships were sitting in a very fragile equilibrium, compatible with the theory of them slowly going down and resting on the bottom of the bay.

When people started growing stuff again, the water was simply brought from where it was to be used to replant and regrow the vegetation.

Could this be a viable theory? I think it could.

  • Not that I'm complaining, but why was EVE, an Earth exploration probe for retrieving plants the single most well-armed robot in the entire ship? Even the security robots (which were probably designed to deal with dangerous rogue robots) didn't have guns and could be beaten up by a nutty robot with oversized fists. So why did EVE have such a powerful blaster cannon? What was she going to encounter on Earth? Menacing trash? Oh, and while I'm at it, why did EVE get stuck to the magnet? Cranes like that have to be electromagnets that can be switched on or off so that things can be dropped, so there should be no magnetism when the thing's powered down. Seeing as it's been broken down for 700 years, I find it doubtful that the thing would be magnetized.
    • As for the magnet, well Wall-E seems to get sidetracked a lot, maybe he fixed it. Why? He's Wall-E, and he's bored as hell.
      • "He's WALL-E, and he's bored as hell" would have made an excellent tagline for this movie.
    • After you turn off any electromagnet, a smaller amount of permanent magnetism remains. Watch any Discovery Channel show with a magnetic crane, you'll notice metal chunks stuck to the lifter when it's shut down. And those are ordinary 21st century technology. A future electromagnet made for cleaning the despoiled Earth should be far more powerful, and would have much greater residual permanent magnetism.
      • EVE herself is magnetic. She can light lightbulbs by touching them, and other neat stuff.
    • I would say the gun is mostly there for Rule of Cool. Also, the weapon may have been installed for demolition purposes, as EVE clearly showed when she got stuck to the magnet. The security bots on the Axiom most likely weren't armed so heavily because they never had to deal with any significant threat. They're mostly glorified traffic cops who are supposed to handle the occasional rogue robot or fallen human.
      • Also, an army of security robots armed with laser cannons would be a phenominally bad idea on a spaceship. All it takes is one stray shot to blow a hole in the hull, and you've created more problems than you've solved. EVE, the only robot designed to work primarily on a planet and not a spaceship, doesn't have that problem.
        • I think it was a specially designed laser (like a projected EMP) that was designed to incapacitate rogue robots without risking damage or injury if misfired. Remember how it was reflected by a parasol?
    • As was said above, not everyone got off the planet. Eve needed the laser to fight off the Crab people Mole People Surviving human population.
    • WALL-E has a laser too - he uses it to cut stuff up into bite size pieces. I assumed that EVE's was just an updated version of his. After all, she would have been stuffed if she hadn't been able to get herself off that magnet, and it'll allow her to get into buildings, fridges, and such.
  • Why did the rogue bots look so happy about destroying all the security bots? Given that all robots seem to be at least somewhat sentient, we see one security bot hide in fear, and EVE seems to have problems disobeying what she's programmed to do, they don't exactly seem inanimate.
    • What, you mean you wouldn't be happy that the massage bot found something else to maim and kill besides you?
    • If we're talking about the same security bot, I don't think it was hiding in fear, I think they threw it in the room and slammed the door. Plus, EVE's directive only overrode her will once, and that was on Earth, where presumably it was a measure to keep the plant safe.
    • And even if the security bots were somewhat sentient, they were still working for AUTO, and after what AUTO did to WALL-E, the proverbial gloves were off.
    • WALL-E kinda seemed to get a "What have I unleashed?!" look at the pile of bodies...
  • How come nobody who spent their entire life staring at a computer screen suffers from chronic, throbbing headaches?
    • Selective breeding and/or genetic engineering. They did have 700 years to get used to it, after all.
    • The holoscreen technology is obviously very advanced.
    • This troper's best guess is headache medicine in the cup-meals.
      • This troper can't remember a time before widespread personal computers and rarely gets these headaches of which you speak. So adaptation.
      • The majority of headaches from computer screens results from low refresh rates. LC Ds and newer technology don't actually refresh the entire screen (only the pixels that change from frame to frame are changed) which removes the flicker problem. The idea of a 'refresh' rate is mostly retained because it makes more sense to people as a unit of measurement.
  • I'll give you that some of the world could slowly devolve into fat blobs but EVERYBODY?
    • They had a pool. So I'm figuring there's some itsy tiny population somewhere that are still more or less human because they were all athletes/joggers/health nuts and taught their children the same. But that would've detracted from the Anviliciousness to show them...
      • The pool and track were relics from the ship's early days as a cruise liner. Nobody had used them in years, they simply were never removed due to the inherent risks in modifying a spaceship while in space. The BnL president himself said that people would have all put on a few pounds after a few years in microgravity, everybody being fat was the natural result of being out there 140 times longer than expected, and thus 140 times the expected bulking up.
    • People are lazy. When given a choice between working, exercising, or sitting around watching TV all day, people will choose the latter.
      • Not true. They will eventually stop watching TV and do other things.
        • Show your working.
          • Paying the power bill?
    • Compare the lifestyle of people today to that of people 700 years ago, or even 100 years ago before mass acceptance of the automobile. Walking 15 miles a day was routine then, it is considered unusual or even cruel today. As soon as any technological advance allows people to get lazier, they get lazier. Doubly so if everyone else is doing it too. WALL-E might make a stretch, but it is not a huge one.
      • According to my research, 15 miles a day in the 1900's was unusual too. Unusual enough for folks to record it and mention how much it was, at least. Closer to 10 miles a day and you still have folks "bragging" about it. Even distances as short as 6 miles (3 there and 3 back) for some folks in countries to the nearest clean water is enough to gain notoriety.
      • Moreover the general pace of a walk hasn't changed much; it still takes someone about 30 minutes at a stroll to go a mile, 20 at a brisk pace, and 10 at a jog. People have just had their distances expanded more; driving 30 miles to go shopping is not that uncommon, but walking for 2 hours to get to school is, when there's no particular need to, outside of someone being unkind.
    • Assimilation, most likely. Like the person above me said, think about all the changes we've had in the past 700 years. Even if there were some people who resented the laziness of the rest of the ship, chances are that the attitude wouldn't have been able to endure for 100 years, let alone 700 in an enclosed and controlled environment like the Axiom. Also, the Health section of the Buy n Large website suggests that the tendency towards lazy lifestyles began long before the Axiom even launched.
  • I was greatly bothered by the scene where the Axiom 'tilts' to the side and all the humans slide into the corner. Doesn't the ship have artificial gravity that would pull you to the 'floor' of the ship, no matter where that floor was? After all, there is no 'up' or 'down' in space. The only way that would make sense would be if the source of gravity on the ship was external, pulling 'down,' like a planet or a star. But there was nothing like that anywhere around the Axiom. It was in the middle of an interstellar gas cloud! So why did the direction of gravity change when the ship tilted?
    • Bugged me too. Probably more than anyone else in the show.
    • It might have been more the momentum of the turn than the turn itself. Remember, the Axiom is a REALLY big ship, and it was rolling pretty fast for such a big object. You know how when a car accelerates really fast you get the sensation of being pushed back into your seat, but when it reaches top speed you don't feel that anymore?
      • But that doesn't make sense. The ship was tilting, not turning.If a ship tilts, the rotation would be about a center axis (as if it was on a seesaw). The people near the edge that 'drops' would fly up into the air, and the people on the edge that 'lifts' would be pressed into the floor, if that was the case. They still wouldn't fall to the corner, unless gravity from outside the ship was pulling them there. Plus, if it was momentum, they would be able to get up after the tilt was done, but they had to wait for the captain to 'right' the ship.
        • The camera obviously stays on the same axis as the ship's artificial gravity. The ship isn't tilting - the camera is.
        • Why would you say the ship was tilting? We never see the outside of the ship in that shot, we have no idea what it was doing.
          • You need to watch the movie again. We do see the outside in that shot, and it was tilting.
          • Perhaps AUTO somehow shifted the artificial gravity to pull everything not bolted down to the side.
          • Once the people start sliding off their chairs, we get a shot outside of the ship where we see it going into roll.
        • Ever been in one of those "Gravitron" machines at the amusement park? It's the same principle, but in reverse.
        • Also, Space Is an Ocean.
    • The ship is not really tilting. It is simply flying in a tight circle instead of straight ahead. The artificial gravity still points "down", but now there is an additional centrifugal "force" acting outward normal to the artificial gravity. From the passengers' perspective, the direction of gravity appears to change. The camera simply tilts to show what gravity feels like to the passengers -- the ship does not really tilt (a meaningless concept in space anyway). Simple high-school physics.
      • When Auto spins his wheel to the right after flinging off the captain, we see a shot of the Axiom executing a positive (starboard) roll, with no evidence of it making a turn. The direction the passengers all start to slide towards is starboard. If centrifugal force was involved, why didn't Pixar just show us that, instead of making it look like the ship was in the influence of an external gravitational field?
      • See Viewers are Morons as well as Rule of Funny.
      • Maybe the gravity was planar and it a set orientation and Auto didn't bother to correct it; this also retroactively explains why at no point anyone is seen walking/floating on the walls or ceiling.
    • The simple fact that the Axoim has artificial gravity of any kind automatically removes it from the realm of the realistic. We can't know how artificial gravity will behave, because we've never invented any. If this specific artificial gravity doesn't move with the floor when the ship tilts, or has to be at exactly 1.0g at all times even when it's inconvenient -- well, maybe that's just the way their Minovsky Physics works.
  • I can't believe this still isn't here, but - in the scene where WALL-E and Eve are both in space - there's no sound in space! Similarly suspicious is WALL-E's ability to survive clinging on to the outside of a spaceship escaping the Earth's atmosphere.
    • As for the sound in space thing, they're robots. They were probably communicating via radio or something. As for WALL-E's remarkable durability, hey, BnL knows how to make 'em to last.
    • It wasn't here because the "there's no sound in space" protest has been made illegal years ago on the grounds of the answer being so offensively obvious. Oh, you didn't know? Well, too late now - please face the wall and stay still while you're arrested.
  • Why go through all the effort of launching a colony spaceship if at first it's intended to just go back to Earth when it's habitable again? Wouldn't underground colonies like Fallout's vaults be easier and more efficient?
    • Cruise ships are more marketable.
    • They may have suspected that it really wasn't possible to clean up the Earth. The space ships were a way to protect humanity in the event of the plan's failure.
      • Indeed, Override Directive A-113 is exactly that. It had to have been thought of before the Axiom left, so that AUTO would know what Forthright was talking about when he invoked it.
  • Why do people assume that EVE was emotionless until she met WALL-E? Didn't they see her zipping around gleefully like an uncaged bird as soon as the spaceship was out of sight?
    • They're used to that plot (boy/girl meets girl/boy with issues, teaches her/him to love) so they see it even when it's not actually really there. Same reason examples get added to tropes where they don't fit in.
    • "People" do? Pro-Mole doesn't. It is quite obvious she has at least some emotions, she just doesn't relate well with other robots. Maybe even she doesn't know how to love indeed.
    • EVE had basic emotions, but she had never interacted with the world the way WALL-E did, likely because of her directive. WALL-E had hundreds of years to himself and plenty of time to explore the concept. EVE appears to have only been sentient when her missions were active, giving her much less opportunity to explore the world beyond her programming. Much like the humans on the ship, EVE isn't emotionless, she just needs something to break her out of the routine she's trapped in so she can learn more about the emotions she has.
    • I think EVE was just easily entertained. But since nobody was around to entertain her until WALL-E came along, it didn't really show.
  • I might have just missed it, but; when EVE is opened in the captain's room for the first time, the plant is absent. Given that she was sealed tight for the entire trip there, how did it escape?
    • EVE was left alone with AUTO's lackey GO-4 before she was taken to the captain.
  • If the Axiom was just some place for the humans to stay while the Earth was being clean up, why did they have to go that far away? Couldn't they have just orbited Earth? It's not as if the space around it would be dangerous. All going so far seems to have accomplished was made it so they couldn't look at Earth, made sending probes much more difficult, and screwing them over if their hyperdrive was broken.
    • Why would it "screw them over" if their hyperdrive was broken? They can fix it. They're entirely self-sufficient after all.
    • It was a cruise. Remember how dirty the shots of Earth proved it to be, even from a distance? Nobody wants to spend their cruise looking at brown on brown.
    • Its possible that they were around, say, Jupiter at first for the above reason, but when the A113 directive came, AUTO took them out farther to reduce chances of anybody noticing.
  • The entire plot of the movie bugged me. The first generation was portrayed as healthier, fitter, and all around better than the last generation, but they were the ones who wrecked the Earth. The fat "lazy" people of the movie were quite industrious, given the chance. Furthermore, it seemed pretty clear that their physiques and behavior were manipulated by the ship to encourage passivity. I know that, being aimed at kids, the lesson was supposed to be "don't be such a couch potato, eat healthy," but as social commentary, it fails utterly.
    • No, the Buy n Large representatives were presented as fit and healthy. They're models, they're supposed to look like that. You don't know what the average person was like in the first generation, and even the president seems like a thoughtless jerk.
      • It seems to me - that most people were just bored of doing nothing. The first human Wall-E encounters complaines of monotony, and the Captain strongly laments 'not getting to do anything'. If the entire human population had spent their life in aimless luxury, I think it's quiite justifyable that they'd be eager to do something more meaningful with their lives, even if they had to get their hands dirty.
    • Also, the movie wasn't intended to have that lesson or be social commentary. It's suppose to be a romantic comedy.
  • There's one thing I don't get about the ending- what, exactly, made Wall-E lose his memory and why did EVE's 'kiss' bring it back? If you think about what she actually did (Specifically, propping him back up with the jack, fixing the treads, replacing an eye, and then replacing the broken board), there's only one thing that could actually wipe his memory, and that's the circuit board. However, this doesn't explain why he still seemed to be himself after getting shocked by AUTO or why True Love's Kiss works.
    • This is Wall-E. Machines are no different from humans. Memory is not stored in memory chips; it's an integral part of the machine soul. It all works better if you consider Wall-E machines to be human brains attached to mechanical parts. That are solar-powered.
    • ...Then how did he lose his memory in the first place?
    • He didn't lose his memory, his eyes weren't properly connected. He did the eye resetting thingy that he did at the start when he replaced his eye. Since he hadn't put the new eyes in himself, he didn't realise that he was blinded. Only the "kiss" made him realise that something was wrong.
    • Amnesia as a side-effect of a near-death experience is not unheard of in fiction.
      • Then why didn't getting zapped cause amnesia?
        • Getting zapped didn't bring back his memory. All it did was cause him to reboot. And this time he managed to get his backups working or whatever.
    • My mom, who happened to be walking through the room at that point on my rewatching the DVD and to whom I commented that this scene had entirely no justification beyond the need for a happy ending, suggested an elegant explanation: He wasn't done rebooting yet. His core programming, aka cleaning up trash, loaded first but it took a long time for the 700 years of memory to load and parse and it probably would've been problematic activating that part of your brain while it was still being worked out, so it wasn't activated until it was done. True Love's Kiss was just a coincidence. This makes more sense when you consider a lot of components, including some internal ones, had just been replaced with unused and presumably factory-new duplicates which had to have stuff written to it, perhaps from a backup.
      • To extend this, WALL-E, almost thoughtfully, as though remembering something important, stops moving completely after EVE holds his hand. I see this as him loading his newer memories last, possibly as a consequence of having linear, futuristic tape-like permanent memory (like his audio recorder), which he loads into more conventional RAM. (also, he stops moving completely after she presses the "Play" button on his recorder. Huge speculation: Considering the repairs she had made, she might have changed the function of the button to "play" the contents of his permanent memory into RAM, thus restoring him.)
    • I don't think WALL-E lost his memory, he just lost his personality. He might recognise EVE as a robot he's previously seen before, but if he's back to his previous factory state, he won't care if she's there and still continue to do his job regardless. The trailers themselves say that WALL-E's personality is a 'quirk' that happened accidentally over 700 years, and thusly WALL-E units aren't supposed to have a personality at all. As to why he temporarily lost it, the 'quirk' might have been buried somewhere when he was rebooted and the Power of Love brought it back.
      • Considering robot personality is apparently contagious, this makes a lot of sense.
    • Well, when EVE kissed WALL-E while they were in space, you can see his reaction could be seen as a small 'system reboot'. When EVE fixed him at the end, he went into a 'safe-mode', and EVE's next kiss made the system reboot again, this time fully.
  • Here's something that bothered me about the whole AUTO issue. Wall-E, EVE, and basically every other machine in the movie is perfectly capable of violating their primary directives whenever they feel like it. Wall-E did so when he abandoned his "clean up Earth" mission to chase EVE. EVE did it when she tried to save Wall-E instead of bringing the plant to HQ. Primary directives are basically suggestions to Wall-E's robots. Except AUTO. Why? Because someone's got to be the villain, and there's nothing more villainous than following orders. * sigh*
    • There's actually a perfectly reasonable Ridiculously-Human Robots explanation for all this. Shelby Forthright, when giving Override Directive A113, says that he's giving the ship over to complete automatic control (more or less). The captain even states that he doesn't get to actually do anything except morning announcements. Since it's pretty clear that robots in WALL-E are basically people, it's safe to assume that they suffer from human flaws... like a desire for power. AUTO writes the plant off as irrelevant because if he didn't, the humans would go back home and he wouldn't have a ship to control anymore.
    • What they said. He's not necessarily incapable of disobeying his prime directive; he just chooses not to. Whether out of malice or stupidity is up to you.
    • Or simply misguided. He isn't what one would call evil-- he IS trying to save humanity from extinction, after all. Take into account his final orders, what had to be 700 years of negative results (even Wall-E had obviously never seen a plant before, and he was ON earth), and the badly degraded condition of the humans aboard both physically and intellectually. AUTO had no reason to believe that a single weed was sufficient proof that earth's ecosphere was survivable. That is one of the best parts about the movie itself--- noone is REALLY a villain; even the worst person there is trying to do the right thing.
      • It's funny how he didn't bother trying to explain any of that to anyone. That might have been the reasonable thing to do.
        • Explained in universe. When the order was given in the second video, it was called a 'super top secret' or something like that, and the captain didn't have authority to ask.
    • Alternately, perhaps the super-override "A113" was simply higher priority than original directives--after all, overrides are intended to stop a robot from doing whatever else it was going to do and make it do what you want instead.
      • This. It may well be that no robot is actually capable of ignoring an override directive - you would expect to have Asimovs three laws or something similar imbedded at a fundamental level, so why not a super-override as well? The reason that the other robots were able to counteract directive A113 is because, owing to its secrecy, it was never actually given to them - Auto was the only robot who had been given a direct order by a duly constituted authority, and since there are more than likely failsafes to prevent robots giving override directives to other robots, he was the only robot who was bound by it.
    • Another theory on this, it has been theorized, and even written about that robots, after a certain amount of time could begin to have their own unique...quirks. Oddities in their programming, evolution on a digital scale. Such as M-O finding himself stuck between two commands, Stay on the white line, and clean up the ship. He breaks one to follow the other. And therefore evolves into more of a character. Remember that everyone of the robots has likely been active for over 700 years, Likely without being shut down or reformatted to ensure that their programming remains pure. Even in Star Wars they comment how droids can pick up odd quirks if they aren't memory wiped every once in a while. By comparison, AUTO was the Auto-pilot, and therefore he would have been re-formatted every day, and re-initialized to make sure he remained in the programming specs. They figured that five years without a wipe would be fine for the lesser robots, and AUTO could take care of them if things got out of hand. But AUTO himself needs to be continously checked, and rechecked. In fact, Wall-E losing his memory returns him (At least temporarily) to his original Programming. Resulting in his returning to factory settings, meaning that it was the fact that he was there for roughly 700 years gave him such an existance, and such a semblance of life.
    • My first thought was that AUTO was trying to prevent humans from recolonizing Earth because that happening would make him obsolete.
    • Some people are like that too. They will follow their orders come Hell or high water. Auto probably didn't consider himself in a position to question his orders.
    • It is possible that the lesser robots, who live in far more variable enviroments (dealing with people, exploring planets) need the ability of disregarding their directives. Dealing with humans, exploring planets, cleaning planets; all are scenarios that contain a ton of unpredictable variables, and the robots need to be able to adapt to those without requiring brand new programming or orders. This could be why some robots go rogue, and why the Security Bots don't grow any kind of personalities. AUTO has fewer variables to consider, and thus sticks much more firmly to his directives - which basically, at the time of the movie, boils down to 1) Keep the Axiom alive and 2) Prevent the Axiom from returning to Earth.
  • What puzzles me isn't why EVE has a powerful weapon/tool. She's being sent into an unknown environment, and the ability to blast things might prove necessary. But why is she so trigger-happy?
    • Her mission is to find plants. Plants don't move very much. Therefore, things that move are fair game a potential threat.
    • Now, be honest. If you had a Lazer Rocket Launcher built into your arm, wouldn't you look for any excuse to fire one off? And to be fair EVE probably had very little hope of finding a Plant after 700 sweeps.
  • Why wasn't the larger trash compacting robots used on earth instead of Wall-E? They would certainly get the job done in less time than Wall-E would have done.....
    • This was asked before. The short version is that smaller robots are better suited to the conditions of Earth; larger ones would have trouble maneuvering and would sink under the massive piles of trash.
      • Ah That would actually make sense! Thanks
  • Why did the Axiom's probe carrier have to land to deploy EVE? She can fly. She can operate in space. And she obviously has plenty of on-board power. The probe carrier could have deployed her capsule from orbit where EVE could have guided the re-entry and discarded the capsule once in the atmosphere. After securing a plant sample EVE could return to orbit before going into beacon mode.
    • The situation on Earth's orbit in Wall-E is pretty bad. There are tens of thousands of sattelites without any control. It means that trash is creating more trash, in turn creating a mess of metal moving at escape velocity which is completely impossible to pass through unharmed. EVE units need an armored ship to ram through that on the way in and out.
    • Did you see how much junk was in orbit? Also, although we do know that EVE can travel in space, we don't know that her on-board energy reserves are sufficient for her to get back into high orbit after an extended survey period. Remember, she shut down everything except standby propulsion and internal climate control when she went into beacon mode.
      • EVE alone would have had an easier time maneuvering through that junk rather than having the probe carrier punch through it twice and risk damaging itself. As for EVE's power reserves, she packs an ion cannon that hits like a small nuke and is deployed at the smallest provocation. She can go supersonic and sustain it without breaking a sweat. And during the chase aboard the Axiom she was flitting all over the place at high speed while carrying WALL-E and blasting away at anything in her path without once stopping for a recharge. Seems to me she's got juice to spare. Going from surface to orbit shouldn't have used more than a fraction of the power she was flaunting throughout the movie.
        • I must disagree. EVE has lots of energy on-board, and her weapon is very high-powered - but it also has a long cycle time for its more powerful shots. Let's say that EVE needs to clear high Earth orbit to get about the debris field (reasonable, since after seven hundred years, orbital decay would have cleared everything but the highest orbitals). If she's of any even marginally reasonable mass, say 25kg, she'd need as much onboard energy as the entire fuel supply of a Titan II at a minimum. It takes enormous amounts of energy to reach high orbit (low orbit is a lot easier).
          • Counter-point. EVE's cannon clearly has its own power source, since it is capable of firing a fully powered blast even when disconnected from EVE's body and its (EVE's) own power source.
          • Depends on how long she takes to achieve orbit. Rockets take a lot of fuel because they reach orbit in a relatively short period of time from launch. Not to mention they have high initial take-off weights. EVE could simply fly while constantly increasing her altitude to the edge of the atmosphere. From there simple orbital mechanics takes over as a low but constant thrust will increase her orbital altitude over time. Her only complication will be jinking to avoid the orbital debris. We've already experimented with using aircraft to carry small rockets to high altitude and then launching them to orbit with much smaller fuel requirements.
            • True, but that only shifts the energy requirements from the rocket to the plane. EVE has no plane or other external vehicle to piggyback on; all the energy she needs to rise over fifty thousand miles has to come from her internal reserves. Those reserves are large, no doubt, but apparently not quite that large. Though why her pod wasn't initially deployed from orbit is beyond me.
    • It's a stretch considering all her other features, but just because you can survive in space doesn't mean you're properly heat-shielded for atmospheric entry/exit. Especially, now that I think about it, while carrying such delicate and important cargo (aka the plant).
      • She would only need heat shielding for re-entry and her capsule could provide that. That's how we deploy planetary probes today.
    • EVE has enough thrust to reach low supersonic speeds (say Mach 1-2), but probably not enough thrust to reach escape velocity of Mach 34. Therefore she needs a big armored ship to get off Earth anyway, so why not have it deliver her too and save her some wear and tear?
      • She doesn't need to reach escape velocity, only orbital velocity. She could place herself in a parking orbit then go into beacon mode. The carrier could then retrieve her without the risks of a planetary landing.
      • She doesn't even need to reach "orbital velocity" the way a rocket does, either. EVE's inertialess drive allows her to hover and fly straight up, and it works just as well in vacuum as it does in atmosphere. There's nothing stopping her from flying straight up until she gets out of the atmosphere. Once she's up there, she can keep accelerating faster and faster until she's in a stable orbit without having to worry about air resistance. And all this without having to achieve the kind of massive accelerations that today's rockets use.
    • Regarding Eve's power reserves, its worth noting that she only does a few bits of flashy flying when she's dropped off, then gets down to her mission. It's not really clear how long she's on earth before Wall-E shows her the plant, but it didn't appear to be more than a few days. Then she rides all the way back to the Axiom (presumably charging along the way), and the Axiom plot takes place over the course of one day. Her energy reserves might not be as big as we're assuming.
    • The simple and straightforward answer is a matter of pragmatism. It saves energy for EVE if she doesn't have to also be equipped to go into space. It might waste energy over all, but if she's expected to survival without any support for unknown periods of time, it's better that she be able to store as much energy as possible and also not have to worry about twice-per-mission equipment that's critical to success. Better to have the probe ship.
  • Related to the above, shouldn't most of that junk have de-orbited by now?
    • Thats the scary part. MOST OF IT ALREADY HAS. That stuff must just be a tiny fraction of what was once there.
    • Incredible, how such a cute film can be High Octane Nightmare Fuel when put with the right Fridge Horror.
  • Also related, the EVE-carrier in question Just Bugs Me. I can understand that it needs to be big to carry the insane amount of fuel required to travel between Earth and the Axiom, but why on Earth... um... I mean, why does it need to land, take off, and then land again? Couldn't it just land once, even assuming that it couldn't park itself in orbit? And why is it clearly a chemical rocket, with massive exhaust plumes that leave a crater of lava on the ground, while EVE clearly has some internal propulsion that allows her to reach supersonic speeds with a tiny engine (most of the space in her is taken up by the plant container, and there must be a battery of some sort too) and whose only waste product is blue lines? This seems so much more efficient, and in space travel, efficiency is pretty high priority.
    • EVE units' trips to Earth normally last years while they scour the entire planet for non-existent plant life -- too much time for her delivery ship to be exposed to the elements and possibly damaged.
  • Why did MOE join Wall-E's cause? I'd think MOE would be programmed to be extremely xenophobic, given his function.
    • Everybody loves WALL-E.
    • To finish cleaning him, that's why.
    • His name is M-O, by the way (it reads in the pod he emerges from).
    • Because he wants to clean up the mess WALL-E and the other robots made. Helping Wall-e is incidental. And perhaps he realizes that the sooner Wall-e goes away, the sooner he can start cleaning.
  • I can't believe this isn't already here. Buy'n'Large made the WALL-Es, the EVEs, and all the other robots. They have figured out how to make a self-sustaining pseudoenvironment inside a spaceship. It's implied on this very page that matter duplication was available long before they left Earth. And yet they couldn't come up with a better waste-management plan back on Earth than "fly off and let a bunch of tiny robots deal with it"? Heck, even putting all the crap into spacefaring garbage trucks and ejecting it into the great beyond would be viable, and we have the technology (if not the resources) for that now.
    • According to the commentary and bonus material on the DVD, the WALL-Es were part of a team. After they compacted the trash, the cubes would be swallowed up by the giant mining robots WALL-E passes on the way home. These robots each had some kind of matter disassembler that would recycle or incinerate the material in the cubes. The effort was abandoned when the WALL-Es finally hit soil and the soil was found to be too toxic. It was judged impractical to dig up large portions of the Earth's soil for cleaning. What technology failed to accomplish, 700 years of sunlight and natural oxidation eventually did.
    • I think it was also mentioned that almost all of the WALL-E units failed to activate properly- a single glitch screwed up the entire operation and nobody got around to fixing it. If things hadn't gone wrong... well, look what WALL-E managed to do all by himself, building an entire cityscape of trash cube skyscrapers. Imagine what a few dozen of him could have done!
      • Except the size of the trash cube piles are meant to show how alone he is and that he has been at it for a LONG time. Combined with that the Axiom was supposed to be in space for only 5 years and even after 700 years it is unclear how effective Wall-E has been. The Wall-Es would have failed because they had no replacement parts (notice how in the storage container/transport Wall-E lives in there is only space for the robots themselves and no extra parts). It makes the Wall-Es seem like a futile gesture to save Earth
  • The one that bugs me is the one that has only been touched on peripherally... forget how could a video tape could last that long, how could ANY of that trash have lasted that long? Anyone who has seen the recent History channel special "Life After People" knows what I'm talking about.... The most stoutly built skyscraper would last approximately 200-300 years. Cars would be heaps of rust in less than fifty. Iron, steel, concrete, wood, paper, plastic..... no material made by man exposed to the elements would last beyond the 400 year mark and most would disintegrate within a century--- not even the toxins in the soil. Even most of the space-trash would have tumbled out of orbit and burned up from reentry. After 700 years, there would literally be no trace whatsoever that human beings had walked the planet--- and that lone little plant would have been growing in the middle of a centuries-old forest. The whole story would have been more feasible if Earth had been a sort of landfill/junkyard/recycling dump for the rest of the solar system, and was "continually" being piled with trash from other planets.
    • In-universe: Buy N' Large products are just that durable (see the cash/coupons still on the floor of the BNL ultra store; a five-year space cruise stretches into 700 with relatively little ill effects on the passengers or "crew"), which is why the earth is still covered in junk. Real life: The director stated that the After Humanity setting was just Rule of Cool and that Act 1 pretty much wrote itself; the dates probably came up when they were writing Act 2. Personally I think it should've been more like a 350-year span, but MST3K Mantra and all that.
    • The biggest culprits for disassembling human structures are other living things. Dirt covers the roads, plants grow and hold it in place. Roots worm their way into cracks in concrete, then expand and break it up. Animals take up residence in skyscrapers, etc. But on Wall-E's Earth, those entities do not exist. They're all dead. So dirt just gets blown away by the next sandstorm. Concrete has to weather away through sheer erosion. From those documentaries, the human structures that will last the longest will be the Pyramids, owing largely to their placement in deserts lacking vegetation. Now imagine the whole world is like that.
      • Which itself leads to more Fridge Logic: Earth's surface remaining even nominally habitable depends entirely on the ecology removing excess gasses like methane and CO2 from the atmosphere. Without photosynthetic life, bacteria and volcanic action would serve to render the atmosphere less and less friendly to life, until finally there's enough to trigger a runaway greenhouse effect.
        • Whether this could happen in a 700-year timespan is up for debate, though. If Earth is well and truly dead, then even decay bacteria are gone and dead biomass won't rot, and thus won't its stored carbon back into the atmosphere. This would slow down the warming process considerably.
        • Long story short, they Did Not Do the Research. No matter how MUCH garbage their was, even with ZERO lifeforms on the planet (and there WERE lifeforms; even cockroaches need an ecosystem), they still had an atmosphere, climate, wind, rain, sun, cold.... Iron rusts, concrete corrodes, plastics and chemicals degrade into their constituent elements (and quite rapidly under ultraviolet light). By 100 years, much less 700, what little hadn't decomposed or corroded or dissolved would have been covered in several hundred feet of sand, silt, loam and clay (aka DIRT.)Which would in turn have been overgrown by plants like the one EVE found. Even at the most generous estimate, Wall-E should have spent the last century or so puttering around in a nascent forest.
    • The A113 Override was sent out in 2110, and by the time Wall-E and EVE reach the Axiom, it's the year 277X. Now, the Axiom has hyperspace capacity, but the ship carrying EVE/Wall-E doesn't. After A113 was recieved, why would AUTO have kept on sending out probes? Sure, inertia and habit make a good argument, but the pilot doesn't seem to really know they even are there, except "they always come back negative"... Couldn't AUTO just stop sending out probes and just not mention them again? I think there could be a simpler explanation to all this. If the 2110 date is 5 years after the end of the 5-year cruise (the first "log" the ship has seems to be from 2100, although that could just be a memory cut-off), then it's possible that AUTO stopped sending out probes a few years afterwards (or perhaps even instantly). However, by that time, the ship has probably gone pretty far. When you've got a hyperdrive, why bother sticking to the Sol System? Why not visit Alpha Centauri, Andromaeda, and a couple other pretty parts of space? If it was intended anything like a cruise, they'd have planned a "sight" every few days. So if they were sufficiently far away, the probe sent in 2109 could have taken 300+ years (even at near-light speed) to reach earth, and just as long (if not more) to get back. I like to think that Wall-E has only been around for 150-200 years (it's circa 2250 when the story starts), but takes 500-550 years to get "back" to the Axiom (possibly because AUTO doesn't really care about the probes and is hopping around using the hyperdrive). It's probably not canon though.
      • This doesn't quite work for a number of reasons: The least of them is that it is implied that the same cockroach is waiting (in what is definitely the same derelict heavy vehicle) for the whole time WALL-E is away. More serious is the fact that the condition of the city when the Axiom lands is not significantly different than it was when WALL-E left. It simply can't be a long time that he's away.
  • This one isn't directly about the film itself... but even disregarding that it wasn't the intended message, why is "We shouldn't cover our planet in garbage and filth until it's uninhabitable" seen as a Left-Wing position, rather than just a generally good idea?
    • Out on a limb, but maybe it's because it's more proscriptive than "Do what you like", which is to say "Buy beautiful things, dispose of them when you lose interest (or something shinier catches your eye) and don't worry about the consequences."
    • In a more cynical sense, because its environmentalism. Conservative naysayers can and do claim that the doomsaying is exaggerated and/or that the costs personally and broadly of changing things for the environmental better are too great for our hardworking businesses to bear- in this case, that the manufacturing industry as it is today relies on such mindless consumerism. Liberals say that this means the system must be changed, of course, but this is about as far as this liberal can take his "devil's advocate" argument. Something about not having the resources to make the change, or the suffering economically during the change being to great, or "I was given large campaign donations by people who stand to gain economically from the status quo remaining", I believe, and I sincerely apologize for my political correctness breaking up at the end there.
    • Because the loudest voices are also the most extremist. For what it's worth, I did read a conservative review of the movie that in turn complained about other conservatives missing the real point of the movie (as below). And I as a moderate conservative get rather pissed off about the extremism and bitching.
    • Because it's been taken over so much by "the Left." There are "Right Wing" environmentalists, but they tend to be rather low key groups and made up of the "wrong" sort of people from the point of view of some of the more extreme "Left Wing" environmentalists. Hunters, mostly (who have a big intensive to preserve wildlife, when you stop to think about it.)
    • As an environmental engineer, it's because this movie is so over the top in it's pollution that I seriously tried to create a "You Fail Environmental Science Forever trope" just for Wall-E and Day after Tomorrow. Waste management doesn't work like that, and pollution has significantly DECREASED in first world countries over the past 50 years. The idea of the world becoming uninhabitable due to pollution is unthinkable in it's impossibility. Unfortunately, people actually believe this is a possible outcome.
  • On a related note to the previous one... how is it that while reading a propagandist message into it that isn't there and plays off of a downright tangential plot element, they seemingly simultaneously manage to completely miss the main thrust of the film, which is essentially "Laziness and running from your problems is bad. Thinking for yourself and taking responsibility is good." I never hear this actually mentioned in discussion of the film, despite how blatant it is, in favor of fabricating an environmentalist message (to condemn) from the premise the main point flows from.
    • When something comes close to a hot-button talking point, people WILL ignore minor details like "plot" and "message" in favor of ranting their side of said hot button talking point. This isn't the first time its happened, and its far from the last time it will happen.
    • Okay, dude, whether the movie was * created* to have an environmentalist message -- I agree it wasn't -- it * does* have an environmentalist message, and ignoring the fact that the message is there is stupid. Most environmentalists would say that "Face your problems rather than being lazy and hoping future generations will do it" * is* the message of environmentalism.
      • The thing is, this is the difference between a sensible environmentalist message and the propaganda message the detractors read into it, which essentially reads "We're all going to die and it's all the fault of big evil corporations and you lazy rednecks, so suck it!" I don't deny there's an environmentalist message -- although it's actually much less central than it's made out to be -- I'm denying that it's the sort of propaganda message they claim.
      • They're living strawmen, nobody cares what they think, despite the fact that they think everybody cares. Like the idiots that call people hippies for wanting to protect the environment, even just a little, and even after doing research.
      • There's a difference between "ignoring the fact that the message is there" and just not giving a fuck. I'd rather enjoy the film, myself
    • To be fair, WALL-E replacing the Pixar light bulb with a "green" light bulb after the credits was a bit groan-worthy. But most of these people probably didn't stay after the credits anyway...
        • So it was a Lampshade Hanging on any Green Aesop it may have?
        • Wha? I think you're seeing things. It looked to me like it was just a regular lightbulb, since the Pixar lamp's bulb had gone out. Just like the regular lightbulb Eve lit up during the movie itself.
        • In this troper's opinion, the complaints from conservatives about the supposed "propaganda" in this movie is incredibly hypocritical. These types of conservatives (it's not all conservatives) won't bat an eye at films like The Chronicles of Narnia, which are blatantly Christian, but when a film comes along that has a message that doesn't conform to their point of view, it's "propaganda". Same thing happened to the Golden Compass, except that was a far less successful film.
  • Robots who follow their programming are bad. Broken robots who spaz out and do things like spray yellow paint all over the floor or spontaneously turn themselves inside out are cute and loveable. The robots who try to correct the defective programming of the aforementioned broken robots so that they stop being inefficient, and even downright dangerous, are also evil. You're part of the machine, man! Break out!
    • The only robot really portrayed as bad is AUTO (and possibly his helper thing), and I'm not sure he was bad because he followed his programming. Wasn't he programmed to allow return to earth eventually, but then decided to ignore that? Anyway, M-O spent half the film vigorously trying to follow his programming and he was a good guy.
      • AUTO was originally programmed to return to Earth, but was given an over-ride command that instead triggered programming to prevent a return by any means necessary. Anyway, the whole point here is to tie into the film's messages about responsibility and thinking for yourself. AUTO is bad not just because he followed his programming, but because he followed it slavishly and without thought, even when conditions changed, refusing to take personal responsibility. As well, the rogue robots are initially portrayed as frightening... but when they decide to rise above their handicaps (and their programming) and help Wall-E, they become heroes. And, as pointed out, robots that follow their programming are portrayed positively when "following their programming" means "living up to their responsibilities", rather than "shirking them because it's easier and less threatening"; M-O is portrayed sympathetically even before he "goes rogue", as is Burn-E from the short of the same name.
    • It's because unlike Auto and his helper thing WALL-E, EVE, M-O and the rejects are considerably "cute" characters. And cuteness determines all ethics.
      • Hey. AUTO and GO-4 are cute. In fact, GO-4 is downright adorable. Heck, AUTO might fall more into the cool than the cute, depending on how you look at it.
  • Why do EVE and WALL-E try so desperately not to get ejected with the garbage? They were in space just a short while ago. They could easily get back in again.
    • EVE was more concerned with protecting WALL-E, who was severely damaged and needed to be tended to immediately. Besides, seeing as that the doors to the Axiom have a bad habit of locking people out (just ask BURN-E), it was probably better she not risk it.
  • I'm surprised a certain subject didn't come up yet - gender roles. That's right. It's obvious Wall-E is supposed to represent a male and EVE a female, with enough Tertiary Sexual Characteristics to keep Viewer Gender Confusion away. But still, in-universe they're a couple of genderless beings sharing platonic love.
    • What precisely "bugs" you about this? If anything, the film helps overturn gender roles by allowing EVE to be the more "dominant" member of the relationship. For one thing, EVE is stronger physically, and carries weapons (both stereotypically "male"). WALL-E is the more "emotional" of the two -- he spends his time cleaning and listening to the Hello Dolly soundtrack (stereotypically "female"). The only thing that makes them seem male and female is the sound of their "voices," and the shape of their bodies (EVE is all sleek, round edges, and WALL-E has a square, boxy frame ... Men have square shoulders/bodies, whereas women have sloping curves). Aside from that, I think they represent very real-seeming personalities that don't fit into prescribed gender-oriented boxes.
      • Yes! Also, EVE is always focused on her directive, while WALL-E would rather just hold hands and stare into EVE's eyes with blurry-love-vision, another reversal of traditional gender roles. Plus, the end of the (awesome) dancing-in-space scene, where EVE gently catches WALL-E in her arms -- both a reversal of traditional gender roles, and a Crowning Moment Of Awww!
    • Well written. This was my favorite aspect of the film.
    • It's probably all in their names as well.
    • But there are other aspects of WALL-E and EVE's characters that were subject to gender roles. Look at what they were designed to do: WALL-E as a somehow-male robot had a manual job with a practical purpose, with a work schedule which enabled him to return home in the evenings, bringing back helpful rewards and have interests beyond his work. EVE as a somehow-female robot was designed to stay in sleep-mode when not directly performing her task, which was basically to find life and incubate it (a pretty good analogy for getting (and staying) pregnant) at which point she, again, went back to sleep-mode until she could "deliver" the plant. Not much of a personality or any initial interests beyond reproductive "work", to the point of literally being a machine (pun not intended) for making babies ahem, growing plants. The fact that she had so much potential for all sorts of work and was obviously over qualified didn't help matters at all, but hey, she was "designed" for a purpose and she was damn well going to see it through to the end! There is even an anti-choice message if one reads way too much into things. And WALL-E taking it upon himself to protect EVE from outside weather forces during the period she was incubating on Earth, when she didn't even need it, was a bit chivalric, wasn't it? None of this really stopped either of them from having their own free will: doing everything in their power to protect each other in actual danger and achieve their goals. However, the Unfortunate Implications are still very much there, begging the question why they even had genders to begin with. They're robots and logically speaking, should have either 1) been gender-neutral and absolutely standardized, or 2) had feminine or masculine personalities come as humanizing aspects of their AI. Since only the latter can be true, why didn't we see other types of mass produced robots that had both male and female members? Instead, certain jobs were simply reserved for "female" robots, such as incubating, care taking and, well, putting on make up; while the majority of all other robots (who did everything from garbage disposal to running a whole fucking self-reliant spacecraft) were voiced by and intended as "male". Now that does invoke gender roles, and it really bugs me.
      • You all but justified your own complaint: why not give a robot for a "feminine" job a feminine design? They were designed by people.
  • Regarding BURN-E - Why were the maintenance ports designed to close the moment ANYTHING passes through them? I mean, anything passing through upon the tracks, I could understand. The ship does need to close them. But what if some debris goes through? A ship the size of the axiom must have SOME gravitational pull- probably enough to bring in any space-debris it passes. So, BURN-E goes out, door remains open, METEOR passes through door, door closes. Now you have damage to the inside of the ship, and one less maintenance droid, as the animation shows they only open from one side. But why not simply open them when something approaches the door on the tracks, close them when they pass through, (or simply go past), and then do the same from the outside? This solves the problem. Or is my grasp of science not right?
    • I think they'd just use the wielding torch to destroy the door (and fix it later) like BURN-E did at the end. Only reason why he didn't consider it before that is because he's silly.
    • The Axiom may be big, but it's also hollow to allow things to move around inside it, so its mass is much lower than you might expect. Space debris also tends to be moving very, VERY quickly, and something as small as the Axiom isn't likely to attract it unless it happens to be moving at relatively the same velocity. That being said, the doors appear to be stupid enough that a meteor that happened to intercept the doors probably would close them.
    • Or the doors are designed to close when an Axiom robot enters it. The designers just never considered the option that any other robot would enter what is obviously a maintenance door (in outer space).
    • How do we know it was automatic? If memory serves, EVE was distracted by carrying WALL-E in her arms at that point. Fly in, hit the Close Door button, get back to business.
  • If the humans have been on the ship for 700 years, wouldn't their immune systems have atrophied? Wouldn't they have died or at least become massively sick once they got back to Earth?
    • AUTO has been taking very, very, very good care of them. Chances are that they were receiving regular vaccines as part of some preprogrammed in passenger maintenance routine intended for if any kids were born on the five year mission, which was then extended to the whole 700 years and many entire lifespans.
    • If enough life on Earth died out, perhaps there just aren't many diseases around any-more by the time the Axiom returns. The germs would have to start developing all over again, right along with the Axiom passengers' immune systems.
      • Another interpretation is that while the ship is tightly controlled its not germ and bacteria free so that people would have immune systems when they got back to Earth.
  • If M-O is an extreme Neat Freak and even went completely berserk after just seeing WALL-E, how is it that he was even able to function at all on the trashed up Earth? His cleaning directive seemed relentless throughout the film until that scene.
    • There's a piece of fanfiction that addresses this. After arriving on Earth, MO is nearly driven crazy by how his scanner sees everything as 'foreign contaminant'. However, he's then convinced by another character, I forgot who, that Earth is not 'foreign', it's 'home', and he understands.
    • This troper also read another piece of fanfiction. M-O keeps his Obsessive Compulsive Directive and he becomes a well established crime detective.
      • Yeah, those are both great, but I was hoping for something a little more Canon.
        • Well, we know that by the end of the film M-O can at least ignore his directive (as he identifies but does not try to clean the plant during the holo-detector scene). It seems M-O was too concerned about WAL L•E's condition to take notice of all the "Foreign Contaminants," even if his warning system was going crazy. Perhaps after the film ended, when he was no longer focused on WAL L•E, it did become aggravating, at which point he had to be reprogrammed.
    • A good explaination is that M-O had two sets of directives; one for the Axiom and one for Earth. On the Axiom, he was programmed to be ruthless about contaminants because it could ruin the delicate quarantine of the ship. Now that he's on Earth, his different set of protocols activate, which would worry more about dangerous contaminants or keeping machinery clean from the elements. The ship was designed to come back, so it makes sense that the robots could be shifted to working elsewhere on Earth.
    • Character Development. He was one of the first 'bots to be able to ignore his directive and something else. He detected the plant as a foreign contaminant, but doesn't try to clean it. Presumably, he can ignore it, or even * select* what qualifies as "foreign contaminant"
  • Okay, here's one I haven't seen addressed yet: Waste Allocating Load Lifter, Earth class; Extraterrestrial Vegetation Evaluator; Microbe Obliterator; A___ U___ T___ O___? Anybody?
    • Auto isn't an Acronym. It's short for Automatic Pilot.
      • Yes, I know, but it just seems to go against all the naming conventions of the robots... except Burn-E, but he was an extra character in a probably non-canon sequence. Unless it's a subtle hint that Auto's 'different'...?
        • Hell, I spent about five minutes sticking words into the letters of Auto's name, and came up with something at least plausible: Axiom class Universal Transportation Organizer... Suck it, Pixar?
  • Why does EVE have eyes? The scene showing how she sees indicates that the black panel on her "face" is a sensor array. Her eyes are simply colored lights on the surface of that array. Also her security camera recordings made when she was in beacon mode were made through the sensor array without her eyes being activated. And since EVE is a probe primarily designed to operate away from humans and human contact, she wouldn't need eyes for the sake of providing a facial focal point for human interaction.
    • All the other robots have eyes and can emote, despite there being, technically, no need for them. Chalk it up to Ridiculously-Human Robots.
    • Maybe to make them look more friendly. Who knows, people might be creeped out if they were told "Hey, don't worry, these things will tell you when the planet is safe!" only to be shown some eyeless EVES.
    • Perhaps EVE didn't originally have eyes, but figured out how to display them herself, and decided to start doing so after gradually developing beyond her original programming, and gaining a real personality, as a result of being around for centuries longer than intended.
    • Alternatively, for the same reason that Pixar designed her that way: the eyes aren't necessary, but they are just so darned cute that her makers slapped them on for no logical reason. All the other benevolent, sentient robots have some unnecessary human characteristics; why not EVE?
    • The EVE models were designed to be humanoid, so that a potential encounter with mutant Mole-People left on Earth would let them bond better with the EVE (and potentially avoid a destruction of the EVE). Contrast the non-human look used for many service bots on the Axiom.
  • When the Axiom probe ship returns to collect EVE it uses massive mechanical arms to manipulate its cargo. The Axiom itself uses similar arms in both the docking bay and the repair ward. However many of the robots are equipped with tractor beams to manipulate their environment. Since these systems seem much more compact and efficient, why not use them for all materiel handling functions?
    • The tractor beams might be much more expensive to make.
    • Alternatively, we never see something as big as EVE moved via tractor beam. Maybe, the cost of producing a tractor beam increases exponentially with the mass limit.
    • Didn't GO-4's tractor beam try to hold onto EVE, and fail only because EVE (with a propulsion system capable of taking her SUPERSONIC, as we see on her first arrival on Earth) was actively fighting him?
    • Tractor beams ARE used to manipulate the ship.
    • Perhaps the tractor beams can only carry/move objects, but not manipulate them or add force to them? It would be quite difficult to handle, say, a screwdriver with a tractor beam.
  • How was the boot with the plant able to fit inside GO-4? It doesn't look like it has much internal space to spare.
    • Gofers ("Go-for") need to be able to carry things. Maybe with a litle brain and salute-widget on top and wheels on the bottom, with carrying space inside?
  • If Humanity had the ability to build city-sized starships capable of sustaining themselves and their passengers for nearly a millennium without human intervention as well as autonomous robots then why was Humanity still confined to Earth? With that technology they could have easily colonized the rest of the Solar System (Buy n Large was planning an outlet mall on the moon) and taken significant steps exploring and colonizing nearby stars.
    • They were planning on starting the colonies really soon, just gotta have this quick five-year cleanup of the old place first! Oops. Might take a bit longer guys, just stay put for now!
  • Less based on the film itself than the audience, it still bugs me that it seems that I'm the only person who happens to be ambivalent toward the movie. It was certainly not bad, but neither did I see it as a masterpiece. It was too cutesy, too idealistic and too preachy for This Troper's tastes. The DVD commentary was what she enjoyed the most, as the creator was just a downright intelligent and interesting guy. However, based on experience, if one comes out and says "I don't think Wall-E is a masterpiece," then the automatic response seems to be "You shallow, idiotic piece of shit. It's people like you who bring humanity down in the direction it takes in the movie!" Why, why, why can't I just not love and not hate it? Seriously, it's just not my kind of movie.
  • Where did WALL-E get a Pong machine? Those things are hard to find now, let alone 700 years from now.
    • Rule of Funny, and MST3K Mantra. A more reasonable explanation would be that Earth-wide culture (or at least the culture of the city WALL-E's cleaning) had an enormous 'Retro-Phase', where old things were studied up, remade, and then sold to the planet. Like how Transformers Animated was basically pseudo-Seventies with most, if not all of their Earth designs. It's not unreasonable that they would've gone so far as to recreate classic video games like Pac-Man and Pong.
    • They're hard for you to find now because you haven't spent seven centuries digging through garbage dumps.
    • He found a video game store that had one. I know at least two video game stores within a half hour's drive that still sell old games, one of them has a whole collection of Atari and Commodore 64 games.
  • The implication among fans that Auto does all the breeding. Considering all the creativity that goes in normal breeding, and how the Internet Is For Porn, I find it very hard that the passenger don't do the horizontal mambo. OK so maybe collection devices are involved but that isn't too far off from a sheet with a hole in it...
    • I think that assumption is driven by the humans' sessile nature. If they ever actually moved anything besides jaws and hands, it would be possible, but considering that they're all functionally glued to their hoverchairs, it is highly improbable that they'd be capable of the acrobatics neccesary to get from one chair onto the person in the other chair without actually walking.
  • At the end EVE has to rebuild WALL-E. All fine and good, except the fact that one of his critical circuitboards is damaged and barely functioning! EVE should've focused on saving his "brain" and then worried about his body! But no, she did it the other way around!
    • He is a robot -- brain and all vital "organs" replaceable. I don't see why reestablishing "brain" fucntion would be as urgent for him as it is for us.
      • Replace the hard drive in your computer with one of the same model. Even though the hardware is now identical, it doesn't work the same without the 'soul' / backups.
      • The damaged circuitboard didn't necessarily have to be the hard drive. It was probably something more like a CPU or motherboard.
  • What does "BURN-E" stand for?
  • So, while Earth was being rebuilt and while they waited for that one little boot seedling to grow, what exactly did those people eat? From what we're shown, their food consists of milkshakes daily. If the Axiom was taken offline (for lack of a better term) when it landed on Earth again, would...whatever the hell it is that makes those food shakes stop functioning? Surely they didn't try turning AUTO back on.
    • I don't see why it would stop. All the other robots on the ship seem to be autonomous, there's no reason the milkshake robots aren't.
    • The credits montage seem to imply that figuring out how to fish, farm and such was a gradual process. The Axiom probably functioned as a combined hotel, hospital and diner during that process.
  • Fridge Logic -- A couple of instances.
    • Some literal Fridge Logic - Why was the (very green) pivotal plant growing inside a closed fridge where no sunlight would be able to get at it? Out of the fridge, understandable, from underneath the fridge, yes, through the top of the fridge, maybe, but this little sapling was very definitely inside the fridge, which WALL-E had to cut open to look inside. Plants kinda enjoy sunlight, don't grow particularly well without it.
      • More on the shoe plant: how did it grow into a giant tree in the credits if it was buried still in the boot?
        • Quite well, apparently. Seriously, it's entirely likely that roots grew out through holes and seams, including the top, where a foot would normally go in.
    • Where were all the other humans? They couldn't have fit all of them on that single ship. The bad dubbing on the BnL CEO's line indicates that the Axiom isn't the only ship they have. However, no other ships returned to earth when EVE puts in the plant. Why? Did AUTO order them to self-destruct? Will they have to take a plant to every ship?
      • Maybe a message was sent to the other ships, but their autopilots kept them in space. After all, the only ship WALL-E was able to mess up was the Axiom.
      • Further supported by the scene where EVE is picked up - and put in a hangar with half a dozen exact replicas.
      • This Troper concluded that the other humans were fine and fit, as it was implied that only the Axiom had the super luxuries.
      • At the end of the movie we see more ships landing in the credits animation. We only see "escape pod" style ships, but they wouldn't be from the Axiom because why would you use the smaller ships to land on earth when you can just walk outside.
      • See What Happened to the Mouse? below...nothing in the movie rules out the possibility that all the other ships came back. The Axiom could simply have communicated to the rest of the BnL fleet that Earth was habitable again.
    • Why would a vegetation scout be equipped both with an itchy trigger finger and a high-powered laser cannon?
      • Presumably in order to take care of situations where it could become trapped, as shown in the movie.
    • If AUTO wanted to make sure that Directive A113 was obeyed, that the ships never returned to Earth, why did it send vegetation scouts in the first place? It had no problem lying to the Captain and going around behind his back to sabotage the mission, so why not just falsify the data and save itself future headaches? Why not sabotage the holo-collector or the sensors of the vegetation scouts so as to never trigger a positive?
      • The Buy-N-Large CEO never actually said not to look for plant life. AUTO, as a robot, probably thought there would BE no plant life ever discovered, because the CEO said the planet was uninhabitable. Also, AUTO was programmed to keep the people placent onboard the Axiom, which required them thinking everything was going to plan.
    • If the "Axiom" routinely dumps several thousand metric tons of garbage into space, where is the material for everything the people on board coming from? Not to mention the steady dumping of air into space: how do the people breathe after several centuries of regular atmosphere dumping?
      • The "Axiom" has to be a closed ecosystem. Plant or bacterial matter onboard could be grown and used to produce oxygen that the humans breathe, the humans then exhaling carbon dioxide that the plants turn back into oxygen, etc. The question of where that mass is coming to make all the stuff they keep dumping off into space is harder to answer.
        • No it doesn't. The scene with the Wall-A's clearly shows them dumping tons of trash overboard. The mass is easy-robotic factories harvesting other planets/asteroids/whatever and shuttling it to the habitat ships. They do have interstellar travel tech AND full artificial intelligence tech. Buy N Large just outsourced a bit.
      • You'd think the huge Axiom-class WALL-A units would be used on the much bigger job of cleaning up Earth. Maybe a horde of WALL-Es was more cost-efficient, or the big ones couldn't be solar-powered. Or BnL just didn't care too much about cleaning up Earth.
        • Addressed above. There is such a thing as "too big", especially when dealing with unstable ground: the A-series can rely on having a solid starship deck under them, while the E-series was designed to cope with shifting trash piles, mud, etc. Lose an E-series to a cave in, oh well. Lose an A-series, that's some expensive gear down the drain!
    • No matter how positive you look at it WALL-E is still going to "die" before EVE, and EVE will be alone
      • Actually, seeing how WALL-E has lasted so long using extra parts, it looks like WALL-E may last longer, seeing how EVE doesn't look like she can be taken apart and fixed (a possible Take That to Apple and its products). That's even more of a Tear Jerker, although on the bright side they don't look like they're going to "die" anytime soon.
    • EVE stands for Extraterrestrial Vegetation Evaluator, but she searches for plants on Earth. So shouldn't she be Terrestrial Vegataion Evaluator?
      • She's from Axiom which is not on Earth so she's extraterrestial.
      • Why not just change the acronym to Earth Vegetation Evaluator? Same name, clearer meaning.
      • Well, since Earth was thought to be eternally uninhabitable, why not scout for other planets that could support life?
    • BURN-E's name indicates he's an Earth-class robot, but by function he's clearly an Axiom-class, and hence should be called BURN-A.
      • The E stands for Engineer.
  • Why hasn't all the garbage decomposed already, cans rust away after fifty years, so after 700, you'd think that nothing would be around.
    • Its known that since air is needed for decomposition, only the top layer in any garbage pile will actually decompose. In this case, evidently the piles of trash were so freaking high that even though there's been 700 years of top layers decomposing to nothing, the stuff that didn't have enough air to decompose remained and remained in volume. In other words, there was Just. That. Much. Trash.
    • Actually, air isn't needed for the decomposition of most organic materials. Metals clearly won't oxidize in the absence of air, but paper, food, grass, leaves, etc. will all happily degrade anaerobically. The Just. That. Much. Trash. argument still works and makes it scarier when you think that those skyscraper piles are that big after settling and decomposition. The bits of paper still around don't make any sense, though.
  • I’m surprised no one else mentioned this. So, the Axiom is big, yeah, but not big enough to house the entire population of Earth. Unless the population of Earth was whittled down to nothing before the departure of the Axiom, there had to been a lot of abandoned people on Earth. Seemed like everyone on the ship was American, dinnit? What happened to the people in Third World Countries, that didn’t have the resources to build a fleet of solar system traversing spaceships? Did other countries/continents send out their own BnL-style ship fleets? How did they decide who got to go on the ship and survive? The moral implications in and of themselves are staggering.
    • It's made clear early in the movie that the Axiom is just one of many ships. You could either be an idealist and say that everyone was able to get on a ship, or you could be more cynical/realistic and say that only people with enough money got in. As for the moral implications, Buy-N-Large is a corporation that's clearly not meant to be seen as "good guys".
      • I don't think Pixar really want to implicate eugenics in their films. I think it's unlikely that people would be denied a place on the escape cruisers on the basis of profit, given that everything is free and no human is expected to lift a finger anyway. I think BnL just provided different ships to different countries. Wall-E landed on a USA cruiser, but it wouldn't be a stretch to imagine a french-speaking cruiser, a mexican cruiser, a fleet of chinese-culture cruisers (and so on). They need humans for profit, so why would they let any of them die? I never got the feeling that Buy n' Large was supposed to be 'evil', their disposition seems to just be 'brainless consumer culture'. I can't see anything truly sinister about them. It's not that kind of film.
      • MR. PRESIDENT! I would not rule out the chance to preserve a nucleus of human specimens...
  • Why does WALL-E freak out when EVE is in maintenance? Earlier in the film, it is shown how he changes his "shoes" and gouges his "eye" out and replaces it. He must have changed his own parts hundreds of times before that, so he should know the difference between robot mutilation and maintenance.
    • Pay more attention to the scene. WALL-E only sees it through a screen, and a combination of what's being done to her, and sounds from another bot being worked on which sound like screams makes WALL-E assume the worst.
  • No one seems to note this, but don't WALL-E and EVE seem a little...big to you? A many a fan artists seem to make them small. Don't they notice WALL-E and EVE are much bigger than that?
    • Take a look at this still [dead link] where WALL-E is holding a Rubik's Cube. Assume that the cube is standard-sized (5.7 cm per side) and compare it to WALL-E. He can't be more than 50 cm tall measuring from the ground to the top of the "eyes". I would call that pretty small.
  • How do the people breathe after getting off the Axiom? I don't think one plant will provide oxygen for an entire planet. And how would there be more types of veggies? I think "plant" would get kinda boring after a while.
    • As mentioned above, they probably sent the EV Es and/or a shuttle over the the Svalbard Doomsday Seed Vault for the seeds to start agriculture anew, and they had the food stores and/or matter converters of the Axiom to last the first few harvests as they get their farmer legs. As for oxygen, nobody'd been using it in 700 years, so enough was left from before everyone left to last until they got plants going, plus I'm guessing that non-edible oceanic algae was making oxygen, probably in increasing quantities over time as the oceanic pollutants wash ashore and/or settle.
    • Firstly, we are shown that there are more plants on Earth than just the one brought to the Axiom. As our closeup view of Earth is pretty much limited to a single city, there might be thriving rain forest elsewhere. Or it progressed to prehistoric times, where oxygen were produced by bacteria rather than plants.
  • Presumably, the reason Wall-E has managed to survive was because he developed a sense of self-awareness. Whereas all the other Wall-Es simply worked until they degraded due to lack of maintenance, the surviving Wall-E was able to identify when he needed repairs and would horde functional parts from 'dead' bots to preserve himself. So my question is, why was Wall-E the only bot to develop a repair instinct, out of the millions that must have been deployed on Earth? Surely they're all manufactured identically, so what allowed this one single unit to adapt where each and every other failed? And furthermore, what kind of AI developer would overlook the critical need for a robot working in Earth's conditions for an indefinite length of time to be programmed with an autonomous repair instict as standard?
    • Like you said, the only source of spare parts for Wall-Es were other Wall-Es. Work it out from here.
    • It was stated that Wall-E was still running because someone missed him when the rest of the Wall-Es were turned off.
    • The WALL-Es were never meant to work for an indefinite amount of time. The plan was to send everyone into space for five years while the Earth was being cleaned (which we all know failed). Considering how confident BnL were that they'd be finished in five years, they didn't plan for the Wall-Es working for a long period of time, or without repair workers on stand-by.
  • What happened to animals?
    • Some would go extinct, while others (like roaches) would survive.
    • Considering that the credits show many Earth species, it's possible that the cruise ships carried specimens with them.
  • Lalala, I'm just going to pretend that AUTO was later turned back on and befriended by WALL-E. Seriously, that fact AUTO - a character that has been determined to be, at most, misguided in his attempt to keep people safe and follow his directive/orders/programming, ends up getting shut off? And never turned back on again? That bugs me, because he clearly has something of a personality, unlike the security bots. GO-4's death also bugged me.
    • Why do you have to pretend? If they are willing to go through the trouble of repairing the rogue robots, why wouldn't they reboot and reformat the Auto-pilot on the 'Jewel of the B n L fleet'? Especially since, after they were settled, humanity would attempt to continue to colonize the moon, explore other planets, and that space cruises could become our equivalent of regular cruises. They would just have to override A113 and update Asimov's laws.
    • Also, there were no parts of GO-4 on the Lido deck when everyone is there for the climax, so there is a very good chance that a cleaning robot picked up the somewhat intact GO-4 (you wouldn't want a very young passenger to hurt themselves). Robots are more elaborate and expensive than our common technology (and apparently very durable), and with such small robots and all those people running around, accidents are bound to happen. So, they would most likely bring the pieces of a robot (versus other trash) to the repair ward before deciding that it was scrap.
  • If the purpose of the Axiom was to be a home-away-from-home while Earth got cleaned up, why did they send it all the way out to a nebula in interstellar space? Couldn't they have done their waiting from, say, Jupiter, or some other conventiently-close place within the solar system with lots of nice raw natural resources?
    • I wasn't aware Jupiter had resources people could use. It's possible that originally they did keep the Axiom closer to Earth, and after waiting 20+ years or so with no changes on their home planet and getting more discouraged each year, people decided they didn't want to keep waking up everyday with their home visible just out a window, still useless. So they (or the Captain) decided "out of sight, out of mind" would help them adjust to life in space more easily, and they went out further into space. A weak argument, I know, but possible.
    • Jupiter consists primarily of hydrogen, and with the matter-energy conversion technology that the Axiom is implied to have, all you really need is hydrogen, and you can create anything from food to a backup copy of the Axiom. As for why the Axiom was out around the Kuiper Belt, it was originally a cruise ship, so it was likely following a preprogrammed course designed for a 'grand tour' of the solar system. With the hyperdrive ability that it demonstrates in the climax, there is no need to stay close to any one planet, since it can be anywhere in the solar system in a matter of seconds/minutes.
  • So if there was so much trash on Earth why didn't they just carpet-bomb the entire surface of the Earth with every bit of ordinance they had on hand? All the thousands of nuclear bombs and numerous conventional bombs would have generated billions of tons of heat and explosive force that would destroyed all the trash on Earth. Given a few hundred years or so the Earth would return within normal livable limits.
    • Because the objective was not any actual, practical, or functional solution to the given problem, but rather to simply guilt-flog humanity in general and western civilization in particular for our "mindless consumerism."
      • Wouldn't the fact that things ever got that bad in the first place be enough to illustrate the point of excessive consumerism being bad? If the reason why everyone was portrayed as so incompetent was simply to illustrate that point then I find myself extremely dissatisfied. There were a bunch of common sense solutions to their trash problems.
    • Because vaporizing the surface of planet doesn't actually fix the problem of making it habitable.
      • Yeah. I think if they had done the "carpetbomb the Earth" thing, they would have pretty much turned it into space rubble.
    • Your plan says people need to stay out of the planet for hundreds of years. BnL original plan was "5 years in space, while cute little robots fix everything". Which one people will take? All and all, BnL plan was better, it's jsut that it failed in way nobody saw coming. Oh, and nuking entire planet would probably kill all life a lot more permanently than trash heaps...
  • where do they get gas to power the Axiom?
    • 1)Mining asteroids 2) Axiom might not need "gas" to power itself.
      • Nebulae are full of lovely resources for a spaceship.
  • So the Captain can read and pronounce the word "acquisition", but he can't pronounce "manual"? Rule of Funny aside, what the hell?
    • Language changes over time. It's reasonable to assume that "acquisition" remained a part of the common vernacular while "manual" fell out of use.
  • In the BURN-E short, while BURN-E is trapped on the outside of the ship, he whiles away the time by using his torch to burn an impression of a flower into the hull. How does he know what a flower is?
    • Basic knowledgeset programmed in and/or he saw holograms of them designed to entertain the passengers.
    • WALL-E came within 50 feet of him. BURN-E is therefore sentient. Besides, if part of his job is to repair lighting units, he probably has been to whatever the Axiom uses for an arboretum and seen flowers while on the job.
  1. Apropos of nothing: It also grew a lot faster than the control plant in the windowsill, and was a lot paler green. Presumably, it was trying to find some light before investing much energy in chlorophyll production.